Telechargé par rubyngan1989

Handbook for Final PE

publicité
INTERNATIONAL SCHOOL OF BUSINESS
Revision on Principle of
Macroeconomics
BY ISB ACADEMIC TEAM
For further information and step-by-step guide to solving
problems, please kindly refer to the Tutoring Videos uploaded on
ISB Academic Team Facebook fanpage.
Under no circumstances should one copy this document without author’s permission.
Chapter 23: Measuring a Nation’s Income
2
Chapter 23:
Measuring a Nation’s Income
I- The Economy’s Income and Expenditure:
- GDP measures two things at once: total income and total expenditure
- Total income = total expenditure (Because money flow from buyer to
seller.)
Chapter 23: Measuring a Nation’s Income
3
II- The Measurement of GDP:
● Definition: Gross Domestic Product (GDP) is the market value of all
final goods and services produced within a country in a given period of
time.
1. “GDP is the Market Value…” : using market prices, because market
prices measure the amount people are willing to pay for different goods,
they reflect the value of those goods.
2. “... of All…” :
+ including all legal product
+ excluding illegal drugs, items produced and consumed at home, never get
into market. (E.g.: Karen cleans her own house instead of hiring people to
clean her house => The value Karen spent cleaning her house is not
included in GDP)
* NOTE: Housing Services.
+ For people who rent their houses, the rent is included in GDP.
+ For people who own their houses, estimated rental value is included in
GDP. (assume that if the owners use their house for rent)
3. “... Final…” :
Chapter 23: Measuring a Nation’s Income
4
+ Final goods are goods purchased to consume.
+ Intermediate goods are goods purchased to produce other goods for sale.
For example:
- A purchases $5 of strawberries to eat. → Then strawberries are
considered as Final goods and $5 will be added into GDP.
- B purchases $5 of strawberries to make a 10-dollar-worth fruit salad to
eat. → In this case, even though strawberries is used to make another
good, B didn’t sale the salad but instead ate it. Therefore, strawberries
are still considered as Final good and $5 is included in GDP.
- C purchases $5 of strawberries to decorate a cake sold in his bakery for
$15. → In this case, strawberries is used to make a cake for sale.
Therefore, strawberries are considered as Intermediate goods and a cake
is a Final good → $15 will be added to GDP.
4. “... Goods and Services…”: tangible goods (food, clothing, car) and
intangible services (haircuts, housecleaning, doctor visits).
5. “... Produced…” : GDP includes goods and services currently produced
and does not includes transactions involving items produced in the past.
For example: Ford produced a $500 car in 2018 but did not sale it until 2019.
➔ $500 is included in GDP of 2018 not GDP of 2019.
6. “... Within a Country…” : within the geographic confines of a country,
regardless of the nationality of the producer.
Chapter 23: Measuring a Nation’s Income
5
For example:
- If a Vietnamese owns a Pho restaurant in the USA, the Vietnamese
owner’s income will be included in the GDP of the USA.
- In contrast, if Ford builds a car factory in Ho Chi Minh city, the
production of the factory will be included in the GDP of Vietnam.
7. “... In a Given Period of Time.” : Usually a year or a quarter (3 months)
The Government reports the GDP for a quarter “at an annual rate” (the amount
of income and expenditure during the quarter multiplied by 4) with seasonal
adjustment.
* NOTE: Another Measures of Income.
GNP = GDP + income nationals earn abroad - income foreigner earn here
= GDP + net transfer (net transfer = cash inflow - cash outflow)
For example:
- If a Vietnamese owns a Pho restaurant in the USA, the Vietnamese
owner’s income will be included in the GNP of Vietnam.
- In contrast, if Ford (a US company) builds a car factory in Ho Chi Minh
city, the production of the factory will be included in the GNP of the
USA.
III- The Components of GDP:
Chapter 23: Measuring a Nation’s Income
6
GDP = C + I + G + NX
With: C is Consumption;
I is Investment;
G is Government Purchases;
NX is Net Exports. (NX = Exports - Imports)
1. Consumption:
Consumption is spending by households on goods and services, with the
exception of purchases of new housing.
For example: My purchases a Pizza for $15 and pays rent for $200.
➔ $215 is included in GDP as consumptions.
2. Investment:
- Investment is the purchase of goods that will be used in the future to
produce more goods and services.
- Including: capital, equipment, inventories, structures, and household
purchases of new housing.
- Investment is different from financial investment (stocks, bonds, and
mutual funds are called saving).
For example: Apple produces $1000 of computers and adds all into inventory.
Chapter 23: Measuring a Nation’s Income
7
➔ Inventory is included in Investment. So $1000 is included in GDP and
counted as Investment.
3. Government Purchases:
- Government Purchases is spending on goods and services by local, state,
and federal governments.
- Including: the salaries of government workers as well as expenditures on
public works.
*NOTE: Social Security benefit to the elderly and Unemployment Insurance
are called transfer payments, they do not produce any goods or services.
Therefore, they are not included in Government Purchases (not included in
GDP).
For example: The government pays the salary of an Army general.
4. Net Exports:
- Net Exports equal the foreign purchases of domestically produced goods
(exports) minus the domestic purchases of foreign goods (imports).
- Domestic buyers buy goods from abroad do not affect GDP.
For example: A Vietnamese buy a Honda car from Japan.
→Net export decreases (Import increases), but Consumption increases by the
same amount → GDP is unaffected.
IV- Real versus Nominal GDP:
Chapter 23: Measuring a Nation’s Income
8
● Nominal GDP: the production of goods and services valued at current
prices. (P x Q) → Not corrected for inflation.
● Real GDP: the production of goods and services valued at constant
prices. (Fixed Pbase x Q) → Corrected for inflation
1. A Numerical Example:
2. The GDP Deflator:
GDP Deflator =
𝑁𝑜𝑚𝑖𝑛𝑎𝑙 𝐺𝐷𝑃
𝑅𝑒𝑎𝑙 𝐺𝐷𝑃
x100
Chapter 23: Measuring a Nation’s Income
9
- At the base year: Nominal GDP = Real GDP
- GDP deflator reflects P level
- GDP Deflator base year = 100
- GDP Deflator > 100 ⇒ inflation
Inflation =
𝐺𝐷𝑃 𝐷𝑒𝑓𝑙𝑎𝑡𝑜𝑟 𝑦𝑒𝑎𝑟 2 − 𝐺𝐷𝑃 𝐷𝑒𝑓𝑙𝑎𝑡𝑜𝑟 𝑦𝑒𝑎𝑟 1
𝐺𝐷𝑃 𝐷𝑒𝑓𝑙𝑎𝑡𝑜𝑟 𝑦𝑒𝑎𝑟 1
x100
V- Is GDP a Good Measure of Economic Well-being?
- GDP doesn’t measure what make life worthwhile, but it measures the
ability to achieve it.
- Things contributing to good life are excluded in GDP. E.g.: leisure.
- GDP excludes value of activity take place out of the market, quality of
environment, distribution of income among the economy.
EXERCISE
1. Which of the following is included in GDP?
A. The sale of stocks and bonds
B. The sale of used goods
Chapter 23: Measuring a Nation’s Income
10
C. The sale of services such as visits to a doctor
D. All of above are correct
2. To compute GDP we
a. Simply sum the number of final goods and services
b. Sum the cost of producing final goods and services
c. Use weights determined by a survey regarding how much people value
different sorts of goods and services to compute GDP as a weighted
average.
d. Sum the market value of final goods and services
3. The simple circular-flow diagram illustrates that
a. Production generates income so that income and production are the same
b. The economy’s income exceeds its production
c. The production of an economy exceeds its income
d. None of the above are correct.
4. The local Chevrolet dealership has an increase in inventory of 25 cars in
2003. In 2004, it sells all 25 cars.
Chapter 23: Measuring a Nation’s Income
11
a. The value of increased inventory will be counted as part of GDP in 2003,
but the value of the cars sold in 2004 will not cause 2004 GDP to
increase.
b. The value of increased inventory will not affect GDP in 2003, but will be
included in 2004 GDP.
c. The value of increased inventory will be counted as part of GDP in 2003
and the value of the cars sold in 2004 will increase 2004 GDP.
d. None of the above are correct.
5. In the U.S real GDP is reported each quarter.
a. These numbers are adjusted to make them measure at annual and
seasonally adjusted rates.
b. These numbers are adjusted to make them measure at annual rates, but no
adjustment for seasonal variations are made.
c. These numbers are at quarterly rates that have been seasonally adjusted.
d. These numbers are at quarterly rates and have not been seasonally
adjusted.
6. Flour is
a. Always counted as an intermediate good
b. Counted as an intermediate good if it is used by a company to make bread
Chapter 23: Measuring a Nation’s Income
c. Counted as a final good if it is used by a consumer who bakes bread for
his own consumption
d. Both b and c are correct.
7. U.S GNP is calculated from U.S GDP by
a. Including income earned by foreigners in the U.S and excluding income
earned by U.S citizens abroad.
b. Including income earned by U.S citizens abroad and excluding income
earned by foreigners in the U.S
c. Including income earned by foreigners in the U.S
d. Excluding income earned by U.S citizens abroad.
8. Greg, a U.S citizen, works only in Canada. The value added to
production from his employment is
a. Included in both U.S GDP and U.S GNP
b. Included in only U.S GDP
c. Included in only U.S GNP
d. Not included in either U.S GDP or U.S GNP
9. If in a given year an economy has consumption of $3000, investment of
$2000, government purchases of $1500, tax of $1200, transfer payments of
12
Chapter 23: Measuring a Nation’s Income
13
$400, depreciation of $300, export of $500 and import of $600. Then GDP
will equal?
A. $6400
B. $7000
C. $8900
D. $9500
10. A firm produces consumer goods and adds some to inventory in the
third quarter of 2018. In the fourth quarter the firm sells the goods at a
retail outlet which leave their inventory diminished. As a result of these
actions, what component(s) of 2019 real GDP change in the fourth quarter?
A. Only investment and it decreases
B. Only consumption and it increases
C. Investment decreases and consumption increases
D. None of the above is correct.
11. Steph buys a designer dress produced by an American-owned fashion
shop in France. As a result, U.S consumption increases, U.S net export
A. Decreases, U.S GDP is unaffected, but U.S GNP increases
B. Decreases, U.S GDP increases, but U.S GNP is unaffected
Chapter 23: Measuring a Nation’s Income
14
C. Decreases, U.S GNP increases, but French GDP is unaffected
D. Are unaffected, U.S GDP is unaffected, but French GDP increases
12. Which of the following is included in the investment component of
GDP?
A. Purchases of stocks and bonds
B. Purchases of capital equipment that were manufactured in a foreign
country by a foreign firm.
C. The estimated rental value of owner-occupied housing
D. None of the above is correct.
13. Transfer payments are
A. Included in GDP because they represent income to individuals.
B. Not included in GDP because they are not payments for currently
produced goods or services.
C. Included in GDP because the income will be spent for consumption.
D. Not included in GDP because taxes will have to be raised to pay for them.
14. What components of US GDP (if any) would each of the following
transactions affect?
a. A family buys a new refrigerator.
Chapter 23: Measuring a Nation’s Income
15
b. Aunt Jane builds a new house.
c. Ford sells a Mustang from its inventory.
d. You buy a pizza.
e. California repaves Highway 101.
f. Your parents buy a bottle of French wine.
g. Honda expand its factory in Marysville, Ohio.
15.
Which of the following statements about nominal GDP and real GDP
is most accurate?
a. Nominal GDP is a better gauge of economic well-being than is real
GDP.
b. Real GDP is a better gauge of economic well-being than is nominal
GDP.
c. Real GDP and nominal GDP are equally good measures of economic
well-being.
d. Whether real GDP or nominal GDP is a better measure of economic
well-being depends on what sort of goods are produced.
16.
Suppose GDP consists of wheat and rice. In 2002, 20 bushels of wheat
are sold at $4 per bushel, and 10 bushels of rice are sold at $2 per bushel. If
Chapter 23: Measuring a Nation’s Income
16
the price of wheat was $2 per bushel and the price of rice was $1 per bushel
in 2001, the base year, nominal 2002 GDP is
a. $100, real 2002 GDP is $50, and the GDP deflator is 50.
b. $50, real 2002 GDP is $100, and the GDP deflator is 200.
c. $100, real 2002 GDP is $50, and the GDP deflator is 200.
d. $40, real 2002 GDP is $100, and the GDP deflator is 50.
17.
The GDP deflator can be used to identify the
a. increase in nominal GDP that is due to an increase in prices rather than
an increase in production.
b. increase in real GDP that is due to an increase in prices rather than an
increase in production.
c. increase in the cost of living for typical U.S. consumers.
d. reduction in government spending required to balance the federal
budget.
18.
Suppose that the country of Wakanda produces only eggs and ham.
The table below shows price of each goods throughout the years. Supposed
that consumers always buy 100 units of eggs and 50 units of ham.
Using 2016 as the base year, calculate the inflation rate.
Chapter 23: Measuring a Nation’s Income
a.
17
Inflation rate in 2017 is 65 percent, inflation rate in 2018 is 4.8
percent
b.
Inflation rate in 2017 is -55 percent, inflation rate in 2018 is 10
percent
c.
Inflation rate in 2017 is -10 percent, inflation rate in 2018 is 5.56
percent
d.
Inflation rate in 2017 is -10 percent, inflation rate in 2018 is -5.56
percent
19.
If a country reported nominal GDP of 100 billion in 2017 and 75
billion in 2016 and reported a GDP deflator of 125 in 2017 and a deflator of
120 in 2016 then from 2016 to 2017
a. real output and prices both rose.
b. real output rose and prices fell.
c. real output fell and prices rose.
d. real output and prices both fell.
Chapter 23: Measuring a Nation’s Income
18
ANSWER AND EXPLANATION
1. C. The sale of services such as visits to a doctor
GDP is the market value of all final goods and services produced .... → used
goods, stocks and bonds are inappropriate
2. d. Sum the market value of final goods and services
(according to definition of GDP textbook page 486-488)
3. a. Production generates income so that income and production are the same
(according to circular-flow diagram, figure 1 textbook page 485)
4. a. The value of increased inventory will be counted as part of GDP in 2003,
but the value of the cars sold in 2004 will not cause 2004 GDP to increase.
Explain: An increase in inventory will be counted as an increase in Investment,
so the 2003 GDP will increase. 25 cars that are sold in 2004 were produced in
2003, so it will not counted in the 2004 GDP.
5. a. These numbers are adjusted to make them measure at annual and
seasonally adjusted rates. (according to PE textbook page 488)
6. d. Both b and c are correct.
Chapter 23: Measuring a Nation’s Income
19
(Explain: Because an intermediate good is a good that is used to make another
good to sell, whereas a final good is a good that is consumed by a consumer for
his own consumption.)
7. b. Including income earned by U.S citizens abroad and excluding income
earned by foreigners in the U.S
(Explain: GNP is the income of all people of that nationality. For example, US
GNP includes income of all American, even abroad. So GNP is GDP including
income earned by US citizens abroad and excluding income earned by
foreigners in the US)
8. c. Included in only U.S GNP
Explain: Since Greg is a US citizen, his employment will be added to the
USA’s GNP. But since he works only in Canada, he will not contribute to the
USA’s GDP
9. A. $6400
(Explain: GDP = $3000 + $2000 + $1500 + $500 - $600 = $6400)
10. C. Investment decreases and consumption increases
(Explain: Because inventory is included in the investment, the diminishing of
inventory means the investment decreases. And when the firm sells the goods at
a retail outlet, the consumption increase.)
Chapter 23: Measuring a Nation’s Income
20
11. A.Decreases, U.S GDP is unaffected, but U.S GNP increases
(Explain: Because the fashion shop is in France, the dress is recognized as an
imported good which leads to the decrease in net export. However, U.S
consumption increases because U.S citizen bought the dress, so the increase in
consumption off set the decrease in net export, which leaves the U.S GDP
unchanged. The owner of the fashion shop in France is American, so U.S GNP
increases.)
12. B. Purchases of capital equipment that were manufactured in a foreign
country by a foreign firm
(Explain: Because the purchases of capital equipment is included in the
investment regardless of where they were made. Stocks and bonds are not
included in the investment, they are included in the saving. The estimated rental
value of owner-occupied housing is included in the consumption, the purchase
of new housing is included in the investment.)
13. B. Not included in GDP because they are not payments for currently
produced goods or services. (according to textbook page 490 part 23-3c)
14.
a. Consumption
b. Investment
Chapter 23: Measuring a Nation’s Income
21
c. Investment, Consumption (Investment decreases, Consumption increases)
d. Consumption
e. Government Purchases
f. Net export, consumption (Buying wine from French affects Consumption
and Import)
g. Investment
15. b. Real GDP is a better gauge of economic well-being than is nominal GDP.
(Explain: Real GDP answers a hypothetical question: What would be the value
of the goods and services produced this year if we valued these goods and
services at the prices that prevailed in some specific year in the past? By
evaluating current production using prices that are fixed at past levels, real GDP
shows how the economy’s overall production of goods and services changes
over time)
16. c. $100, real 2002 GDP is $50, and the GDP deflator is 200
Chapter 23: Measuring a Nation’s Income
Year
Price of
Quantity of
wheat
bushels of
22
Price of rice
Quantity of
bushels of rice
wheat
2001
$2
1
$1
1
2002
$4
20
$2
10
17. a. increase in nominal GDP that is due to an increase in prices rather than
an increase in production. (according to textbook page 494)
18. d. Inflation rate in 2017 is -10 percent, inflation rate in 2018 is -5.56
percent
Chapter 23: Measuring a Nation’s Income
23
19. a. real output and prices both rose
Compute the Real GDP, we got 2016 Real GDP = 62.5, 2017 Real GDP = 80
Since Real GDP and GDP Deflator both increase from 2016 to 2017, Price level
and real output increase.
Chapter 24: Measuring the Cost of Living
24
Chapter 24:
Measuring the Cost of Living
I/ The consumer price index (CPI):
1/ How the CPI is calculated:
CPI: a measure of the overall cost of the goods and services bought by a
typical consumers.
The CPI increases => consumers need to pay more to maintain the same
living standard.
Formula:
𝐶𝑃𝐼 =
𝑃𝑟𝑖𝑐𝑒 𝑜𝑓 𝑏𝑎𝑠𝑘𝑒𝑡 𝑖𝑛 𝑐𝑢𝑟𝑟𝑒𝑛𝑡 𝑦𝑒𝑎𝑟
× 100
𝑃𝑟𝑖𝑐𝑒 𝑜𝑓 𝑏𝑎𝑠𝑘𝑒𝑡 𝑖𝑛 𝑏𝑎𝑠𝑒 𝑦𝑒𝑎𝑟
Price of a basket is calculated as the total price of all the goods in the basket.
𝐶𝑃𝐼 𝑦𝑒𝑎𝑟 2 − 𝐶𝑃𝐼 𝑦𝑒𝑎𝑟 1
𝐼𝑛𝑓𝑙𝑎𝑡𝑖𝑜𝑛 𝑟𝑎𝑡𝑒 𝑖𝑛 𝑦𝑒𝑎𝑟 2 =
× 100
𝐶𝑃𝐼 𝑦𝑒𝑎𝑟 1
Chapter 24: Measuring the Cost of Living
25
* Producer Price Index: measure cost of basket for producers.
2/ Problems with CPI:
1. Substitution bias:
Suppose there are 2 goods in the basket, A and B. When price of A
increases more rapidly than price of B. Consumers will be more likely to buy
less A and more B. CPI ignores this change in quantity bought because the
basket is fixed.
2. Introduction of new good:
Consumers have greater set of choice from which to choose => Less
dollars are needed to maintain the same level of well-being => Each dollar
becomes more valuable. Because CPI has fixed basket, it does not reflect to
increase in dollar value.
3. Unmeasured quality change:
People try to measure quality in terms of price, but it is hard to measure it
correctly when goods’ quality changes over time.
● All 3 problems with CPI above overstate the CPI (and in turn Inflation
Rate)
26
Chapter 24: Measuring the Cost of Living
3/ CPI vs GDP Deflators:
a. What is included?
The first difference is that the GDP deflator reflects the prices of all
goods and services produced domestically, whereas the consumer price index
reflects the prices of all goods and services bought by typical consumers.
GDP Deflator
CPI
Imported goods
✗
✓
Capital goods
✓
✗
Imported goods are not included in the GDP because when people buy
imported goods, the increase in Consumption offsets the decrease in Net Export.
CPI, on the other hand, will include Imported goods if it is typically bought by
consumers.
27
Chapter 24: Measuring the Cost of Living
Capital goods are included in the GDP (if it is produced domestically)
because it is represented by the increase in Investment. CPI, on the other hand,
does not include Capital goods because it is not typically bought by consumers.
E.g.: Purchase of domestic industrial tractors (capital good) is included in
GDP because it is produced domestically but not in the CPI because a typical
consumer does not buy tractors.
b. Differences:
GDP Deflator
CPI
Fixed prices from base year
Fixed basket from base year
G&S produced domestically
G&S bought by a typical consumer
Include Capital goods (if domestic)
Exclude Capital goods
Exclude Imported goods
Include Imported goods (if typically
bought by consumers)
II/ Correcting economic variables for the effects of inflation:
28
Chapter 24: Measuring the Cost of Living
1/ Dollar figures in different times:
Value of dollars is different in different times due to changes in inflation.
Formula:
𝑇𝑜𝑑𝑎𝑦 $ = 𝑌𝑒𝑎𝑟 𝑇 $ ×
𝑃𝑟𝑖𝑐𝑒 𝑙𝑒𝑣𝑒𝑙 𝑡𝑜𝑑𝑎𝑦
𝑃𝑟𝑖𝑐𝑒 𝑙𝑒𝑣𝑒𝑙 𝑖𝑛 𝑦𝑒𝑎𝑟 𝑇
The Price Level can be determined by CPI.
E.g.: Determine how much does an amount of 2018 $500 worth in 2019,
today. Given that the CPI in 2018 is 169, CPI in 2019 is 225.
𝑇𝑜𝑑𝑎𝑦 $ = 𝑌𝑒𝑎𝑟 2018 $ ×
𝑃𝑟𝑖𝑐𝑒 𝑙𝑒𝑣𝑒𝑙 𝑡𝑜𝑑𝑎𝑦 (2019)
𝑃𝑟𝑖𝑐𝑒 𝑙𝑒𝑣𝑒𝑙 𝑖𝑛 𝑦𝑒𝑎𝑟 2018
= $500 × 225
169
= $665.68
2/ Indexation:
Indexation: Auto correction of a dollar amount for the effect of inflation
E.g.: Cost-of-living allowance (COLA) automatically changes when CPI (and
Inflation Rate) changes.
Chapter 24: Measuring the Cost of Living
29
3/ Real vs Nominal Interest Rate:
Formula:
Real interest = Nominal interest - Inflation
* Real interest: how fast your purchasing power increases, corrected for
inflation
* Nominal interest: how fast your money increases, not corrected for
inflation
*Note: 3 most important categories of consumer’s spending:
- Housing
- Transport
- Food and beverage
Chapter 24: Measuring the Cost of Living
EXERCISE
01/ True or False
a. Suppose that the USA import cameras from Canada. An increase in the
price of imported cameras is captured by the USA’s CPI but not by the
USA’s GDP deflator.
b. An increase in the price of helicopters purchased by the UK armed
forces is captured by the CPI.
30
Chapter 24: Measuring the Cost of Living
31
c. The "base year" in a price index is the benchmark year against which
other years are compared.
d. If the CPI rises at 5 percent per year, then every individual in the
country needs exactly a 5 percent increase in their income for their
standard of living to remain constant.
e. The largest category of goods and services in the CPI is food and nonalcoholic beverages.
02/ An inflation rate calculated using the CPI shows the rate of change of
a. all prices.
b. the prices of all final goods and services.
c. the prices of all consumer goods.
d. the prices of some consumer goods.
03/ Which one of these is a problem arisen from the measurement of CPI?
a. When price of good A rises faster and price of good B, people will buy
less A and more B
b. When a new good enters the market, it is not counted in the CPI
c. Increased quality are hard to be measured in terms of price
32
Chapter 24: Measuring the Cost of Living
d. All of the above
04/ The following table shows the prices and the quantities consumed in the
country known as the University States. Suppose the base year is 2003. This
is the year the typical consumption basket was determined so the quantities
consumed during 2003 are the only quantities needed to calculate the CPI
in every year.
Year
Price of
Quantity
Price of
Quantity
Price of
Quantity
Books
of Books
Pencils
of
Pens
of Pens
Pencils
2003
€50
10
€1
100
€5
100
2004
€50
12
€1
200
€10
50
2005
€60
12
€1.50
250
€20
20
a. What is the value of the CPI in 2003?
b. What is the value of the CPI in 2004?
c. What is the value of the CPI in 2005?
d. What is the inflation rate in 2004?
e. What is the inflation rate in 2005?
Chapter 24: Measuring the Cost of Living
33
f. What type of bias do you observe in the CPI and corresponding inflation
rates you generated above?
05/ Suppose that you lend your flatmate $100 for one year at a nominal
interest rate of 9%
a. How many dollars of interest will your flatmate pay you at the end of the
year?
b. Suppose at the time you both agreed to the terms of the loan, you both
expected the inflation rate to be 5 per cent during the year of the loan.
What do you both expect the real interest rate to be on the loan?
c. In the case described above, actual inflation turned out to be higher than
expected. Which of the two of you had the unexpected gain or loss? Your
flatmate (the borrower), or you (the lender)? Why?
d. What would the real interest rate on the loan have been if the actual
inflation rate had turned out to be 11 per cent?
e. Explain what it means to have a negative real interest rate.
f. Suppose that after 1 year, you have already paid back your loan and
interest. A few days later he complained that your $109 can buy 4% less
good than before. What is the inflation rate?
Chapter 24: Measuring the Cost of Living
34
06/ Andrew is offered a job in Des Moines where the CPI is 80 and a job in
New York where the CPI is 125. Andrew’s job offer in Des Moines is for
$42,000. How much does the New York job need to pay in order to make
the two salaries have almost the same purchasing power?
07/ If your wage rises from €500 per week to €625 per week while the CPI
rises from 112 to 121, how should you feel about your standards of living? .
08/ 20 years ago, a typical accountant earned a salary of $2000. What
should the salary of a typical accountant now in order to have the same
purchasing power as before? Given that the CPI in 2019 is 148 and the CPI
in 1999 is 80.
ANSWER AND EXPLANATION
1.
A. True. Explain: Camera is a good typical consumers buy, so it’s in the basket
and an increase in price leads to an increase in CPI. On the other hand, the
camera is imported from Canada, so it increases Consumption but decreases
Net Export, so it is counted in USA’s GDP Deflator.
B. False. Explain: Helicopter is not the kind of goods a typical consumers
would buy so it does not show up on the CPI
C. True.
Chapter 24: Measuring the Cost of Living
(Explain: Theoretical Information)
D. False
(Explain: When the CPI rise by 5% per year, every individual in the country
need
5%
increase in their income for their standard of living to
𝐶𝑃𝐼 𝑖𝑛 𝑦𝑒𝑎𝑟 1
remain constant)
E. False. (Solutions: Housing and Transportation (see p.508, PE text book for
further details))
2. D
Explain: Because the CPI calculated the price of the goods in the basket, so
only some of the goods, not all of them
3. D (Explain: Theoretical Information)
4.
A. (Answer: 100)
B. (Answer: 145.45)
C. (Answer: 250)
D. (Answer: 45,45%)
E. (Answer: 71.88%)
35
Chapter 24: Measuring the Cost of Living
36
F. (Answer: Substitution bias since the price of pen rises faster than the price of
other two goods, people will buy more books and pencils and less pens)
5.
A. ( = 100 * ( 1 + 9%)) = $109)
B. (Answer: Real i = Nominal i - Inflation = 9 - 5 = 4%))
C. (Answer: Borrower gains, Lender loses)
D. (Answer: Real i = Nominal i - Inflation = 9 - 11 = - 2%)
E. (Answer: You will receive more in terms of money, but in terms of purchasing
power, you receive less)
F. ( Answer: Inflation=13%)
6. (Answer: Applying the formula of Inflation, we can see that the Price level in
New York is 56.25% greater than one in Des Moines.
NY’s Salary = 42000 * ( 1 + 56.25% ) = $65,625)
7. (Answer: % increased in wage = 25%
Inflation = 8.03%
So Wage rises more than the price level => Increased standards of living)
8. (Answer: Using the formula of exchanging money
2019 salary = 2000 * (148 / 80) = $3700)
Chapter 25: Production and Growth
37
Chapter 25:
Production and Growth
I/ Determinant of Productivity:
a/ Physical Capital (Capital): the stock of equipment and structures that are
used to produce goods and services.
E.g.: To produce a table, the workers needs equipment like saws, drills, etc. So
drills, saws, etc. are Physical Capital
b/ Human Capital: the knowledge and skills that workers acquire through
education, training, and experience.
E.g.: skills accumulated throughout the life of a worker are called Human Capital.
c/ Natural Resources: the inputs into the production of goods and services that
are provided by nature, such as land, rivers, and mineral deposits.
E.g.: To build a hydropower plant, people need a fast-current river. So the river
is Natural Resource.
Chapter 25: Production and Growth
38
d/ Technological Knowledge: society’s understanding of the best ways to
produce goods and services.
*Note:
- Technological Knowledge and Human Capital are closely related but
distinct.
- To use a relevant metaphor, technological knowledge is the quality of
society’s textbooks, whereas human capital is the amount of time that the
population has devoted to reading them.
EXERCISE
1/ What are the main determinant of productivity:
a. Physical capital, human capital, natural resources, and
technological knowledge.
b. Government purchase, human capital, natural resources and
technique.
c. Physical capital, human capital, natural resources and consumption.
d. Physical capital, quantities demanded, natural resources and
technological knowledge.
2/ Choose the correct answer:
(i) Human Capital refers to workers’ knowledge and skills
Chapter 25: Production and Growth
39
(ii) Technological Knowledge and Human Capital is exactly the same
(iii) The builder use saw, hammer,... to build a house. Saw, hammer,.. are
Physical Capital.
(iv) The jeweler uses ruby ore to make a ruby ring. The ruby ore is
Physical Capital
a. (i), (ii)
b. (i), (iii)
c. (ii), (iv)
d. (iii), (iv)
3. Which of the following statements is WRONG:
a. Technological knowledge is the quality of society’s textbooks, whereas
human capital is the amount of time that the population has devoted to
read them.
b. Although technological knowledge and human capital are closely related,
there is an important difference.
c. To evaluate the economy’s efficiency, economists base mostly on
technological knowledge rather than human capital.
d. Natural resource are both renewable and nonrenewable.
Chapter 25: Production and Growth
40
ANSWER AND EXPLANATION
1. a. Physical capital, human capital, natural resources, and technological
knowledge.
(Explain: Theoretical Information)
2. b. (i), (iii)
(Explain:
(i) is correct. Human Capital definition
(ii) is wrong. Technological Knowledge is closely related to Human Capital, but
they are different. To put it simply, technological knowledge is the quality of
society’s textbooks, whereas human capital is the amount of time that the
population has devoted to reading them.
(iii) is correct. Saw, hammer,... are tools used to build a house, so they are
Physical Capital.
(iv) is wrong. Ruby ore is provided by nature so it is Natural Resources.
3. c. To evaluate the economy’s efficiency, economists base mostly on
technological knowledge rather than human capital.
Chapter 25: Production and Growth
41
(Explain:
A, B is correct. To put it simply, workers accumulate knowledge and skill
(Human Capital) through society understanding (Technological Knowledge)
D is correct. Theoretical Information.
C is wrong. To evaluate the economy efficiency, we cannot base simply solely
on Technological Knowledge and Human Capital but all four factors including
Physical Capital, Human Capital, Natural Resources, and Technological
Knowledge)
Chapter 26: Saving, Investment, and the Financial System
42
Chapter 26:
Saving, Investment, and the Financial System
I/ Bond:
- IOU (I owe you); certificate of indebtedness
E.g.: A buys a bond from B. B will pay A periodically according to
interest rate until the bond matures. When the bond matures, B will repay A the
amount of money A had provided initially.
Terminology:
- Date when the full amount will be paid: date of maturity
- Amount borrowed initially: principal
- Length of time until bond matures: term
- Bond that never matures: perpetuity
- Probability the borrower will fail to repay: credit risk
Chapter 26: Saving, Investment, and the Financial System
43
- Such failure: default
- Sales of bond: debt finance
- Owner of bond: creditor
- Long term is riskier than short term, so long-term interest rate is higher
than short-term interest rate.
iLT > iST
- Financially shaky firms issue high interest rate bond.
- Stocks and bonds are store of value.
II/ Stock:
A claim to partial ownership of a firm
- Sales of stock: equity finance
- Stock pays depend on profit of the firm
- Stock: higher risk but also higher return potential
- Stock price reflect profitability
- Bond holders’ payment is included in company’s expense.
Optimistic about company’s future → Demand for stock increases →
Price of stock increases
Pessimistic about company’s future → Demand for stock decreases →
Price of stock decreases
● How bonds and stocks payment are made:
Chapter 26: Saving, Investment, and the Financial System
44
● Formula:
+ Earnings = Retained Earnings + Dividend
+ Dividend (per share) = Dividend yield * Stock price per share
+ Price-Earning Ratio = Stock Price per share / Earnings per share
III/ Bank:
Take deposit from who want to save, use deposit to lend to people, charge
borrower higher interest rate on their loans.
Banks are:
- Medium of exchange
- Store of value (like stocks and bonds)
IV/ Mutual fund:
Sell shares to public, use proceeds to buy stocks and bonds.
- Advantages:
Chapter 26: Saving, Investment, and the Financial System
45
+ Small amount of money can diversify their holding (Economists
are sure about this)
+ Give access to professional money managing skills (Economists
are skeptical about this)
V/ Saving and Investment in the National Income Accounts:
For a closed economy (does not trade with other countries):
Saving = Investment
For an individual, the equation above might not hold. But as a closed
economy,Saving always equals Investment.
S = I = (Y - T - C) + (T - G)
S: national saving
(Y - T - C): private saving
● Note: Private saving must always be Y - T - C, NOT Y - C - T
(T - G): public saving
T>G: budget surplus
T<G: budget deficit
● Note: “Saving” and “Investment”
+ Saving: When someone earns more than spends, he/she will use his
excess income to put into banks or buy stocks or bonds. That action
is Saving.
Chapter 26: Saving, Investment, and the Financial System
46
+ Investment: Investment refers to the purchase of new capital, such
as new equipment or buildings.
VI/ Market for Loanable fund:
- Supply comes from savers (Savers supply funds)
- Demand comes from investors (Investors borrow funds)
- Real interest rate: amount borrowers pay for their loans and savers
receive.
a/ Saving incentive:
Reduce tax on interest income => Savers now will get more interest than
before => Save more => Supply curve shift right.
b/ Investment incentive:
Chapter 26: Saving, Investment, and the Financial System
47
An investment tax credit gives advantage to any firms buildings new
factory or buying new equipment => Investors will borrow more money to buy
capital => Invest more => Demand curve shift right
c/ Budget Surplus and Deficit:
Budget deficit: Supply curve shifts left (Crowding-out Effect: A
decrease in investment that results from government borrowing)
Budget surplus: Supply curve shifts right.
EXERCISE
1.
Which of the following statements is most likely to be correct?
a. A general, persistent decline in stock prices is a signal that the economy
is about to enter a boom period because people will be able to buy stock
for less money.
b. A general, persistent decline in stock prices is a signal that the economy
is about to enter a recession because low stock prices may mean that
people are expecting low corporate profits.
c. A general, persistent decline in stock prices does not tell us anything
about the business cycle because stock prices can fall for many reasons.
d. A general, persistent decline in stock prices is a signal that the economy
is about to enter a recession because low stock prices mean that
corporations have had low profits in the past.
Chapter 26: Saving, Investment, and the Financial System
2.
48
PacknCamp Corporation has price per 1 share of stock of $50, has
issued 2,000,000 shares of stock, has retained earnings of $2 million dollars,
and a dividend yield of 2 percent. The price-earnings ratio of PacknCamp
is
a. 45.
b. 50
c. 15
d. 25
3.
Suppose that in a closed economy GDP is equal to 10,000, taxes are
equal to 2,500 Consumption equals 6,500 and Government expenditures
equal 2,000. What are private saving, public saving, and national saving?
Does the government run into budget deficit or surplus?
a. 1500, -1000, 500, budget surplus
b. 1000, 500, 1500, budget surplus
c. 500, -1500, 1000, budget deficit
d. None of the above are correct.
4.
The country of Hykania does not trade with any other country. Its
GDP is $20 billion. Its government purchases $3 billion worth of goods and
services each year, collects $3 billion in taxes, and provides $2 billion in
Chapter 26: Saving, Investment, and the Financial System
49
transfer payments to households. Private saving in Hyrkania is $4 billion.
What is investment in Hykania?
a. $4 billion
b. $3 billion
c. $2 billion
d. There is not enough information to answer the question.
5.
The Eye of Horus incense company has $10 million in cash which it
has accumulated from retained earnings. It was planning to use the money
to build a new factory. Recently, the rate of interest has increased. The
increase in the rate of interest should
a. not influence the decision to build the factory because The Eye of Horus
doesn’t have to borrow any money.
b. not influence the decision to build the factory because its stockholders are
expecting a new factory.
c. make it more likely that The Eye of Horus will build the factory because
a higher interest rate will make the factory more valuable.
d. make it less likely that The Eye of Horus will build the factory because
the opportunity cost of the $10 million is now higher.
6.
If there is shortage of loanable funds, then
Chapter 26: Saving, Investment, and the Financial System
50
a. the supply for loanable funds shifts right and the demand shifts left.
b. the supply for loanable funds shifts left and the demand shifts right.
c. neither curve shifts, but the quantity of loanable funds supplied increases
and the quantity demanded decreases as the interest rate rises to
equilibrium.
d. neither curve shifts, but the quantity of loanable funds supplied decreases
and the quantity demanded increases as the interest rate falls to
equilibrium.
7.
Suppose that the government were to replace the income tax with a
consumption tax. This would make the interest rate
a. and Quantity of Loanable Fund increase.
b. and Quantity of Loanable Fund decrease.
c. increases and Quantity of Loanable Fund decreases.
d. decreases and Quantity of Loanable Fund increases.
8.
In the last few years the U.S. government has gone from a surplus to
a deficit. Other things the same, this means that
a. supply of loanable funds shifted right.
b. supply of loanable funds shifted left.
c. demand for loanable funds shifted right.
Chapter 26: Saving, Investment, and the Financial System
51
d. demand for loanable funds shifted left.
9.
On February 1st, 2019, Company A issued a short-term bond of
$100000, the amount borrowed would be re-paid completely on February
1st, 2020. What is the principal and the term of the bond, respectively? Is
the Interest Rate more likely to be low or high?
a. 1 year, $100000, High Interest Rate
b. $100000, 1 year, Low Interest Rate
c. The principal cannot be determined without the Interest Rate, 1 year, Low
Interest Rate
d. 2019, 2020, High Interest Rate
10.
Which of these is a Store of Value?
a. Stock
b. Bond
c. Bank Deposits
d. All of the above
11.
Stock in Synergistic Corporation is selling at $25 per share. It had
earnings of $5 a share and a dividend yield of 5 percent. What is the
dividend and the price-earnings ratio?
a. $.25, 5
Chapter 26: Saving, Investment, and the Financial System
52
b. $.25, 6.7
c. $1.25, 5
d. $1.25, 6.7
12.
If Microsoft sells a bond they are
a. borrowing directly from the public.
b. borrowing indirectly from the public.
c. lending directly to the public.
d. lending indirectly to the public.
13.
Other things the same, as the maturity of a bond becomes longer, the
bond will pay
a. less interest because it has less risk.
b. less interest because it has more risk.
c. more interest because it has more risk
d. There is no relation between term to maturity and risk.
14.
All else equal, when people become more optimistic about a
company's future, the
a. supply of the stock and the price will both rise.
b. supply of the stock and the price will both fall.
Chapter 26: Saving, Investment, and the Financial System
53
c. demand for the stock and the price will both rise.
d. demand for the stock and the price will both fall.
15.
Henry buys a bond issued by Ralston Purina, which uses the funds to
buy new machinery for one of its factories.
a. Henry and Ralston Purina are both investing.
b. Henry and Ralston Purina are both saving.
c. Henry is investing; Ralston Purina is saving.
d. Henry is saving; Ralston Purina is investing.
ANSWER AND EXPLANATION
1. b. A general, persistent decline in stock prices is a signal that the
economy is about to enter a recession because low stock prices may mean
that people are expecting low corporate profits.
(Explain: Stock’s price is determined by people’s perception of the
corporation’s future profitability.
So low stock price suggests that people are expecting low corporate profit.)
2. d. 25
(Explain: Dividend= Dividend yield * Price
Chapter 26: Saving, Investment, and the Financial System
54
= 2% * 50 = $1
Retained Earning of EACH stock = $2000000 / 2000000 = $1
=> Price-Earning Ratio = Price / (Dividend + Retained Earning) = 25)
(NOTE: Earnings is DIFFERENT from Retained Earning. Earning = Retained
Earning + Dividend )
3.b. 1000, 500, 1500, budget surplus
(Explain: Private Saving = 10000 - 2500 - 6500 = 1000
Public Saving = 2500 - 2000 = 500
National Saving = 10000 - 6500 - 2000 = 1500
The government is running Budget Surplus )
4. c. $2 billion
(Explain: Note: From a macroeconomics standpoint, Transfer Payment is a
Negative Tax
Private Saving = Y - T -C
= $4 billion
→ C = $15 billion
Closed economy → Investment = $2 billion)
Chapter 26: Saving, Investment, and the Financial System
55
5. d. make it less likely that The Eye of Horus will build the factory because
the opportunity cost of the $10 million is now higher.
(Explain: The opportunity cost of building a new factory with $10 million is
giving to the bank and earn Interest. When Interest Rate increased, the Interest
amount you get from the bank is now higher.)
6. c. neither curve shifts, but the quantity of loanable funds supplied
increases and the quantity demanded decreases as the interest rate rises to
equilibrium.
(Explain: When there is a shortage of loanable fund, Borrowers will demand
less and Savers will supply more. As a result, the market eventually gets back to
its equilibrium.)
7: d. decrease and investment increase.
(Explain: Consumption tax encourages Savings, the Supply curve for the
market of loanable fund will shift to the right.)
8. b. supply of loanable funds shifted left.
(Explain: Government finance budget deficit by borrowing in the bond market.
The Supply curve of the market for loanable fund is determined by National
Saving, which is the sum of Private Saving and Public Saving. When the
governments run a budget deficit, Public Saving decreases → National Saving
Chapter 26: Saving, Investment, and the Financial System
56
decreases → Supply for loanable fund decreases → The Supply curve shift to
the left)
9. b. $100000, 1 year, Low Interest Rate
(Explain: The principal is the amount borrowed. The term is the length of time
until the bond matures. The bond will be more likely to have a Low Interest
Rate because it is Short-term.)
10. a. All of the above
(Explain: theoretical information)
11. c. $1.25, 5
(Explain: Dividend = $1.25
Price-Earning Ratio = 25 / 5 = 5)
12: a. borrowing directly from the public.
(Explain: Selling/Buying bonds a type of Financial Market that directly
provides fund. When Microsoft sells a bond to the public, the money flow from
the public to Microsoft, so they are borrowing money from the public.)
13. c. more interest because it has more risk
(Explain: Long-term bond is more riskier than short-term bond so it has higher
interest rate)
14. c. demand for the stock and the price will both rise
Chapter 26: Saving, Investment, and the Financial System
57
(Explain: When people are optimistic about a company profitability, they
demand more shares of stock, thus increase the price)
15. d. Henry is saving; Ralston Purina is investing.
(Explain: Section 26-2b, p. 556, PE textbook
One is considered saving when he is contributing to the national saving,
Investment, on the other hand, is buying factors of production to produce goods
or even produce more factors of production.)
58
Chapter 28: Unemployment
Chapter 28:
Unemployment
I- Identifying Unemployment:
1. How is Unemployment Measured?
- 3 groups of population:
Employed: paid employees, self-employed, and unpaid workers in a
family business. People who have jobs but are temporarily absent.
Unemployed: people who are not working, have looked for work during
the previous 4 weeks. Waiting to be recalled from having been laid off.
Not in the labor force: everyone else who does not fit into the first 2
categories. Including full-time students, retirees, and homemakers.
=> Labor force: the total number of workers, including the employed
and unemployed.
Labor Force = number of employed + number of unemployed
Unemployment rate: the percentage of the labor force that is
unemployed.
59
Chapter 28: Unemployment
Unemployment rate=
𝑈𝑛𝑒𝑚𝑝𝑙𝑜𝑦𝑒𝑑
𝐿𝑎𝑏𝑜𝑟 𝐹𝑜𝑟𝑐𝑒
× 100%
Labor-force participation rate: the percentage of the adult population
that is in the labor force
Labor-force participation rate=
𝐿𝑎𝑏𝑜𝑟 𝐹𝑜𝑟𝑐𝑒
𝐴𝑑𝑢𝑙𝑡 𝑃𝑜𝑝𝑢𝑙𝑎𝑡𝑖𝑜𝑛
× 100%
Natural Rate of Unemployment: The normal rate around which the
unemployment rate fluctuates
Cyclical Unemployment Rate: The deviation of unemployment rate
from its natural rate.
2. Does the Unemployment Rate Measure What We Want It
to?
The u-rate is not a perfect indicator of joblessness or the health of the
labor market:
❏
It excludes discouraged workers (Individuals who would like to
work but have given up looking for a job)
❏
It does not distinguish between full-time and part-time work, or
people working part time because full-time jobs not available.
❏
Some people misreport their work status in the BLS survey.
Chapter 28: Unemployment
However, the u-rate is still a very useful barometer of the labor market &
economy.
3. Why Are There Always Some People Unemployed?
+ Frictional unemployment:
- Occurs when workers spend time searching for the jobs that best
suit their skills and tastes
- Short-term for most workers
+ Structural unemployment
-
Occurs when there are fewer jobs than workers
- Usually longer-term
II- Job Search:
Job search is the process of matching workers with appropriate jobs.
1. Why Some Frictional Unemployment Is Inevitable?
- Frictional unemployment is often the result of changes in the demand
for labor among different firms
- Sectoral shifts are changes in the composition of demand across
industries or regions of the country
Due to Sectoral Shift, some workers need to quit an industry and
move to another. However, it takes time for them to find new jobs =>
Sectoral Shift creates Frictional Unemployment.
60
Chapter 28: Unemployment
61
2. Public Policy and Job Search:
- The faster information spreads about job openings and worker
availability, the more rapidly the economy can match workers and firms,
the lower the Frictional Unemployment rate.
3. Unemployment Insurance:
- Unemployment insurance (UI): a govt program that partially protects
workers’ incomes when they become unemployed
- UI increases frictional unemployment because the reduce job search
effort.
III- Minimum-Wage Laws:
The minimum wage may exceed the equilibrium wage => Surplus of
workers => Unemployment for workers with least skills.
Chapter 28: Unemployment
62
IV- Unions and Collective Bargaining:
1. The Economics of Unions:
- Union: a worker association that bargains (collective bargaining) with
employers over wages, benefits, and working conditions. If bargaining is
not successful, they strike (organize large-scale withdrawal of labor)
- Unions exert their market power to negotiate higher wages for
workers.
2. Are Unions Good or Bad for the Economy?
- When unions raise the wage above equilibrium => surplus of labor =>
unemployment results.
=>
Chapter 28: Unemployment
63
+ “Insiders” – workers who remain employed, are better off
+ “Outsiders” – workers who lose their jobs, are worse off
- Some outsiders go to non-unionized labor markets, which increases
labor supply and reduces wages in those non-unionized markets.
V- The Theory of Efficiency Wages:
● Efficiency Wages: above-equilibrium wages paid by firms to increase
worker productivity.
● Type of Efficiency-wage theory:
- Worker Health:
+ In less developed countries, poor nutrition is a common problem.
+ Paying higher wages allows workers to eat better, makes them
healthier, more productive.
- Worker Turnover:
+ Hiring and training new workers is costly.
+ Paying high wages gives workers more incentive to stay, reduces
turnover.
- Worker Quality:
Offering higher wages attracts better job applicants, increases
quality of the firm’s workforce.
Chapter 28: Unemployment
64
- Worker Effort:
+ Above-equilibrium wage makes workers more eager to keep their
jobs => have more incentive to work, not to shirk.
EXERCISE
1.
Cyclical unemployment refers to
a. the relation between the probability of unemployment and a
worker’s changing level of experience.
b. how often a worker is likely to be employed during her lifetime
c. year-to-year fluctuations of unemployment around its natural rate.
d. long-term trends in unemployment.
2.
Who would be included in the labor force?
a. Derrick, who is waiting for his new job to start
b. Brett, who has become discouraged looking for a job and has quit
looking for awhile
c. Julia, an unpaid homemaker
d. None of the above would be included in the labor force.
3.
Which of the following definitions is correct?
Chapter 28: Unemployment
65
a. Labor force = number of employed.
b. Labor force = population – number of unemployed.
c. Unemployment Rate = (number of unemployed / [number of
employed + number of unemployed]) x 100.
d. Unemployment Rate = (number of unemployed / adult population)
x 100.
4.
In 2000 in the United Kingdom, the adult population was about 46.5
million, the labor force participation rate was 63.5 percent, and the
unemployment rate was 5.8 percent. What was the number of people
employed and the number of people unemployed?
a. about 29.5 million and 2.7 million.
b. about 29.5 million and 1.7 million.
c. about 27.8 million and 2.7 million.
d. about 27.8 million and 1.7 million.
5. The BLS recently reported that there were 48.6 million people over age
25 who had at least a bachelor’s degree. Of this number, 38.0 million were
in the labor force and 36.9 million were employed. What was the laborforce participation rate and the unemployment rate for this group?
a. about 97 percent and about 2.9 percent
Chapter 28: Unemployment
66
b. about 97 percent and about 2.3 percent
c. about 78 percent and about 2.9 percent
d. about 78 percent and about 2.3 percent
6.
Suppose that some people are counted as unemployed when, to
maintain unemployment compensation, they search for work only at places
where they are unlikely to be hired. If these individuals were counted as out
of the labor force instead of as unemployed,
a. both the unemployment rate and labor-force participation rate
would be higher.
b. both the unemployment rate and labor-force participation rate
would be lower.
c. the unemployment rate would be lower and the labor-force
participation rate would be higher.
d. the unemployment rate would be higher and the participation rate
would be lower.
7.
There will be structural unemployment if
a.
some wages are kept above the equilibrium level.
b.
some people choose not to work at the equilibrium wage.
Chapter 28: Unemployment
8.
c.
some wages are below the equilibrium level.
d.
All of the above could be correct.
67
Frictional unemployment can be the consequence of
a. workers leaving existing jobs to find ones they like better.
b. an industry declining while another is growing.
c. Government support a period amount of money to the unemployed.
d. All of the above are correct.
9.
Tom is looking for work after school, but everywhere he fills out an
application he is told that so have lots of others. Simon has a law degree.
Several firms have made him offers, but he thinks he might be able to find
a firm where his talents could be put to better use.
a. Tom and Simon are both frictionally unemployed.
b. Tom and Simon are both structurally unemployed.
c. Tom is frictionally unemployed, and Simon is structurally
unemployed.
d. Tom is structurally unemployed, and Simon is frictionally
unemployed.
Chapter 28: Unemployment
10.
68
Consumers decide to buy more computers and fewer typewriters. As
a result, computer companies expand production while typewriter
companies lay-off workers. This is an example of
a. structural unemployment created by efficiency wages.
b. cyclical unemployment created by a recession.
c. frictional unemployment created by a sectoral shift in demand.
d. None of the above are correct.
11.
Which of the following is correct?
a. Unemployment insurance raises structural unemployment because
it reduces the job search of the unemployed.
b. When the government lift off minimum wage law, Structural
Unemployment increases.
c. A typical unemployed worker near the end of the time in which she
can receive benefits increases her job search efforts.
d. All of the above are correct.
12.
Which of the following would be associated with the type of
unemployment created by the minimum wage law?
Chapter 28: Unemployment
69
a. Greg quits his job as a grocery cashier because he would rather
work at the library.
b. Fred decides to close his Thai restaurant because he cannot afford
to pay workers the going wage.
c. In hopes of earning higher income Bruce goes back to trade school
to get further training.
d. Liz pays the workers for her plumbing and heating company more
than the equilibrium wage because she believes it will make them
work harder.
70
Chapter 28: Unemployment
Figure 1
13.
Refer to Figure 1. If the minimum wage is $5, the number of people
unemployed is
a. 40 and if the minimum wage is $6 it is 0.
b. 20 and if the minimum wage is $6 it is 20.
c. 40 and if the minimum wage is $6 it is 20.
d. 0 and if the minimum wage is $6 it is 0.
14.
Refer to Figure 1. If the minimum wage fell from $7 to $5
a. unemployment would fall by 20.
b. unemployment would fall by 40.
Chapter 28: Unemployment
71
c. unemployment would be unchanged.
d. unemployment would rise by 20.
15.
Unions
a. do not increase the natural rate of unemployment.
b. do not raise the wages of workers in the union.
c. raise the profits of firms they work for.
d. cause wages of non-unionized workers to fall.
16.
Mary Sue is the newly appointed CEO of a company that
manufactures CD drives on an assembly line. Her staff has told her that the
output the firm produces, given the number of workers employed, indicates
that some workers may be shirking. According to efficiency wage theory,
what should she do?
a. pay all workers more than the equilibrium wage rate
b. pay all workers below the equilibrium wage rate to make up for the
loss from shirking
c. make sure that workers are getting paid exactly the equilibrium
wage rate
Chapter 28: Unemployment
d. reduce production
17.
The natural unemployment rate includes
a. both frictional and structural unemployment.
b. neither frictional nor structural unemployment.
c. structural, but not frictional unemployment.
d. frictional, but not structural unemployment.
18.
The natural rate of unemployment
a. is a constant.
b. is the desirable rate of unemployment.
c. cannot be altered by economic policy.
d. None of the above are correct.
19.
Which of the following would reduce the natural rate of
unemployment?
a. The Internet provides more readily available information about
available jobs.
b. Congress increases the minimum wage.
c. Laws prohibits agencies to help transitioning workers from
declining to growing industries.
72
Chapter 28: Unemployment
d. All of the above are correct.
20.
If an unemployed person quits looking for work, the unemployment
rate
a. decreases, and the participation rate increases
b. decreases, and the participation rate decreases
c. stays the same, and the participation rate decreases
d. and the labor force participation rate stay the same.
21. Theory of Efficiency Wage suggests that above-equilibrium wage:
A. Increases workers’ health, quality, and effort and decreases turnover
B. Decreases workers’ health, quality, and effort and increases turnover
C. Is a waste of resources because it has no effect on workers.
D. Cannot create Unemployment.
ANSWER AND EXPLANATION
1. c. year-to-year fluctuations of unemployment around its natural rate.
(Explain: Theoretical Information)
73
Chapter 28: Unemployment
74
2. a. Derrick, who is waiting for his new job to start.
(Explain:
A is correct. Derrick is included in the labor force because he has a job, he just
hasn’t started it yet.
B is wrong. Brett would like to work but has given up finding a job. So Brett is
not included in the labor force.
C is wrong. Julia is an unpaid homemaker. So she is not included in the labor
force (See PE textbook p.587))
3. c. Unemployment Rate = (number of unemployed / [number of employed +
number of unemployed] x 100.
(Explain: Theoretical Information)
4. d. about 27.8 million and 1.7 million.
(Explain:
Since 63.5% of the population join the labor force, Labor Force = 63.5% x 46.5
million = 29.5 million.
Unemployment Rate = 5.8%.
Unemployed Population = Unemployment Rate x Labor Force = 5.8% x 29.5
million = 1.7 million.
75
Chapter 28: Unemployment
Employed Population = Labor Force - Unemployed Population = 29.5 million 1.7 million = 27.8 million)
5. c. about 78 percent and about 2.9 percent
(Explain:
Labor Force participation rate =
38 𝑚𝑖𝑙𝑙𝑖𝑜𝑛
48.6 𝑚𝑖𝑙𝑙𝑖𝑜𝑛
× 100% =
Unemployed Rate =
𝐿𝑎𝑏𝑜𝑟 𝐹𝑜𝑟𝑐𝑒
𝐴𝑑𝑢𝑙𝑡 𝑃𝑜𝑝𝑢𝑙𝑎𝑡𝑖𝑜𝑛
78%
𝑈𝑚𝑒𝑚𝑝𝑙𝑜𝑦𝑒𝑑
𝐿𝑎𝑏𝑜𝑟 𝐹𝑜𝑟𝑐𝑒
𝐿𝑎𝑏𝑜𝑟 𝐹𝑜𝑟𝑐𝑒 − 𝐸𝑚𝑝𝑙𝑜𝑦𝑒𝑑
𝐿𝑎𝑏𝑜𝑟 𝐹𝑜𝑟𝑐𝑒
=
× 100% =
× 100% =
× 100%
38 𝑚𝑖𝑙𝑙𝑖𝑜𝑛 − 36.9 𝑚𝑖𝑙𝑙𝑖𝑜𝑛
38 𝑚𝑖𝑙𝑙𝑖𝑜𝑛
× 100% =
2.9%)
6. b. both the unemployment rate and labor-force participation rate would be
lower.
(Explain:
When people are counted as out of the Labor Force instead of Unemployed.
76
Chapter 28: Unemployment
Labor-Force participation Rate would be lower because there are now less
people in the Labor Force
Unemployment Rate would be lower because there are now less unemployed
people)
7. a. some wages are kept above the equilibrium level.
(Explain: Structural unemployment means that there are not enough position for
the demand for job. This is cause by Wages kept above Equilibrium, causing a
Shortage for jobs
)
8. d. All of the above are correct.
Chapter 28: Unemployment
77
(Explain: Frictional Unemployment is when the Unemployed are looking for
suitable jobs. This could be caused by: Job Search, Sectoral Shift,
Unemployment Insurance.)
9. d. Tom is structurally unemployed, and Simon is frictionally unemployed.
(Explain: Tom has to compete with each others for one position, which
indicates that there are not enough positions for all the workers finding jobs =>
Tom is structurally unemployed
Simon is currently finding a position which fits his skills the most => Simon is
frictionally unemployed.)
10. c. frictional unemployment created by a sectoral shift in demand.
(Explain: Due to the change in need of consumers, computers companies
expand production while typewriter companies have to lay-of workers. This
illustrates Sectoral Shift - a change in composition of demand among industries.
Workers leaves the Typewriters companies to find more suitable jobs in
Computer companies => They are frictionally unemployed until they have
found a new job)
Chapter 28: Unemployment
78
11. c. A typical unemployed worker near the end of the time in which she can
receive benefits increases her job search efforts.
(Explain:
A is wrong. Unemployment Insurance raises Frictional Unemployment .
B is wrong. When the government lift off minimum wages law, Structural
Unemployment will decrease.
C is correct. When the Unemployment Insurance is running out of time,
the unemployed will increase his/her job search effort)
12.b. Fred decides to close his Thai restaurant because he cannot afford to pay
workers the going wage.
(Explain:
A is wrong. No information about Wage is given.
B is correct. Minimum wage above equilibrium causes Unemployment.
C is wrong. Bruce is not unemployed, he is currently out of the labor
force.
D is wrong. Liz is practicing the law of Efficiency Wages, nothing relate
to Minimum wages.)
Chapter 28: Unemployment
79
13. d. 0 and if the minimum wage is $6 it is 0.
(Explain: The minimum wages of $5 is currently less than the equilibrium wage
of $6. So the price floor has no effect on the market of labor. So market is still
at equilibrium => No unemployment
Same goes for minimum wage of $6)
14. b. unemployment would fall by 40.
(Explain: When the Minimum Wage is $7
Unemployment = Excess Supply - Short Demand = 80 - 40 = 40
When the minimum wages fall to $5, it has no effect on the market because it is
lower than the equilibrium wages => Market goes back to the equilibrium =>
No Unemployment => Unemployment falls by 40)
15. d. cause wages of non-unionized workers to fall.
(Explain:
A is wrong. Unions use collective bargaining to ask for higher wages =>
Causing Unemployment
B is wrong. Unions use collective bargaining to ask for higher wages.
80
Chapter 28: Unemployment
C is wrong. Unions use collective bargaining to ask for higher wages =>
Reduce quantity of labor of a firm. So the firm now has less labor and has to
pay higher wages => Less profit.
D is correct. When workers are fired from unions, they move to a nonunionized industries => Increase Supply of Labor in non-unionized industries
=> Lower wages.
)
16.a. pay all workers more than the equilibrium wage rate.
(Explain: Some workers are not working at their full potential. According to
Efficiency Wage theory, the CEO should increase salaries to encourage the
productivity of workers.)
17. a. both frictional and structural unemployment.
(Explain: Theoretical Information)
Chapter 28: Unemployment
81
18. d. None of the above are correct.
(Explain: Natural Rate of unemployment:
- Always fluctuates
- Is just the normal rate of unemployment around which the unemployment
rate fluctuates, not a desirable rate
- Can be altered by public policy like Minimum Wage Law)
19. a. The Internet provides more readily available information about available
jobs.
(Explain:
A is correct. The Internet makes it easier for Job Search.
B is wrong. Minimum wage only creates Unemployment.
C is wrong. It will be harder and longer for an unemployed to find a new
job without the help of agencies)
20: b. decreases, and the participation rate decreases
(Explain: When an unemployed person quits looking for jobs, he is now a
discouraged worker, which is now counted as out of the labor force. So Laborforce Participation rate decreases.
Chapter 28: Unemployment
82
Since Unemployed is a sub-category of Labor Force, a person who is out of the
Labor Force is now out of the Unemployed category, so Unemployment rate
decreases.)
21.A
(Explain: Theoretical Information
D is wrong because Efficiency Wage can also cause Unemployment like
Minimum Wage law)
Chapter 29: The Monetary System
83
Chapter 29
The Monetary System
I- The Meaning of Money:
Money: the set of assets in an economy that people use to buy goods and
services from other people.
1. The Functions of Money:
3 functions of money:
+ Medium of exchange: an item that buyers give to sellers when they want
to purchase goods and services.
+ Unit of account: the yardstick people use to measure prices and record
debts.
+ Store of value: an item that people can use to transfer purchasing power
from the present to the future.
Liquidity: how easy it can be to transfer to medium of exchange.
2. The Kinds of Money:
2 kinds of money:
+ Commodity money: money with intrinsic value.
Chapter 29: The Monetary System
84
E.g.: gold, cigar
+ Fiat money: money without intrinsic value, used as money by
government decree.
E.g: Dollars, Vietnam Dong.
3. Money in the U.S. Economy:
+ Currency: the paper bills and coins in the hands of the public.
+ Demand deposit: balances in bank accounts that depositors can access on
demand by writing a check.
❖ 2 measures of the money stock for the U.S. economy:
- M1: Currency, Demand deposits, Traveler’s check, Other checkable
deposits (more liquid).
- M2: M1 + Savings deposits, Small time deposits, Money market mutual
funds, A few minor categories (less liquid).
Chapter 29: The Monetary System
85
II- The Federal Reserve System:
+ Federal Reserve (Fed): the central bank of the United States
+ Central Bank: an institution designed to oversee the banking system and
regulate the quantity of money in the economy.
+ Money Supply: the quantity of money available in the economy.
+ Monetary Policy: the setting of the money supply by policymakers in the
central bank.
Chapter 29: The Monetary System
86
III- Banks and the Money Supply:
- 100-percent-reserve banking: not influence money supply
- Fractional-reserve banking: a banking system in which banks hold only
a fraction of deposits as reserves. → when banks hold only a fraction of
deposits in reserve, the banking system creates money.
For example:
First National Banks
Assets
Liabilities
Reserves
$10
Loans
$90
Deposits
$100
* Note: Assets = Liabilities.
➢ Before First National Bank makes any loans, the money supply is the
$100 of deposits.
➢ When the bank lends out some of the deposit, the borrowers hold $90 in
currency.
➢ The money supply = Currency + Deposits = $100 + $90 =$190
Chapter 29: The Monetary System
87
➔ Fractional-reserve banking system creates money but not wealth,
because people are still in debt.
+
Reserve ratio (R) =
𝒓𝒆𝒔𝒆𝒓𝒗𝒆
: the fraction of deposits that banks hold
𝒅𝒆𝒑𝒐𝒔𝒊𝒕𝒔
as reserves. (Reserve = Required reserve + Excess reserve).
- Suppose the system consists of countless banks and the process of
creating money goes on and on:
+ Money multiplier: the amount of money the banking system generates
with each dollar of reserves.
→ Money multiplier = 1/R
→ R ↗ => money multiplier ↘
For example: $100 is deposited into the fractional-reserve banking system with
the reserve ratio of 5%. How much money does the banking system generate?
SOL:
𝛥𝑀𝑜𝑛𝑒𝑦=𝛥𝐷𝑒𝑝𝑜𝑠𝑖𝑡 × 𝑀𝑜𝑛𝑒𝑦
𝑚𝑢𝑙𝑡𝑖𝑝𝑙𝑖𝑒𝑟=
❖ A More Realistic Balance Sheet:
𝐷𝑒𝑝𝑜𝑠𝑖𝑡
𝛥
𝑅𝑒𝑠𝑒𝑟𝑣𝑒 𝑟𝑎𝑡𝑖𝑜
=
$100
5%
= $2000.
Chapter 29: The Monetary System
88
Assets
Liabilities
Reserves
Deposits
Loans
Debt
Securities (stock, bond)
Capital (owners’ equity)
+ Leverage: the use of borrowed money to supplement existing funds for
purpose of investment
+
𝑎𝑠𝑠𝑒𝑡𝑠
Leverage ratio =
𝑐𝑎𝑝𝑖𝑡𝑎𝑙
E.g.: Leverage ratio = 20 => For every dollar the owner contributed, the bank
has $20 of assets, $19 are financed with borrowed money by taking in deposits
or issuing debt.
+ Capital requirement: minimum amount of bank capital
IV- The Fed’s Tools of Monetary Control:
1. How the Fed influence quantity of reserve:
a. Open-market operations (OPO):
- Is the method the Fed uses most often.
Chapter 29: The Monetary System
89
→ buy bond => ↗ money supply
→ sell bond => ↘ money supply
b. Fed lending to banks:
- Banks borrow from discount window and are charged with discount
rate.
+ When discount rate ↗ =>Banks will borrow less => Money Supply ↘
+ Discount rate ↘ => Banks will borrow more => Money Supply ↗
2. How the Fed influence Reserve Ratio:
a. Reserve requirement:
- The Fed rarely uses this method.
- The regulation that set the minimum amount of reserves that banks must
hold.
+ R ↗ => Money Multiplier ↘ => Money Supply ↘
+ R ↘ => Money Multiplier ↗ => Money Supply ↗
b. Interest on reserve:
Chapter 29: The Monetary System
90
- The Fed will pay banks interest depends on the amount of reserves they
hold.
+ Interest ↗ => R ↗ => Money Supply ↘ ; Money Multiplier ↘
+ Interest ↘ => R ↘ => Money Supply ↗ ; Money Multiplier ↗
3. Problems in controlling Money Supply:
● Fed doesn’t control the amount households choose to hold in deposit at
the bank.
● Fed doesn’t control the amount bankers choose to lend.
=> Cannot perfectly control money supply
4. Federal Fund Rate:
- A bank lends out to other banks and charges interest.
- How the Fed controls Federal Fund Rate:
+ Sell bonds => Federal Fund Rate ↗ => Reserve ↘ => Money Supply ↗
+ Buy bonds => Federal Fund Rate ↘ => Reserve ↗ => Money Supply ↘
EXERCISE
1/ If you deposit $100 into a demand deposit at a bank, this action by itself
Chapter 29: The Monetary System
91
a. does not change the money supply.
b. increases the money supply.
c. decreases the money supply.
d. has an indeterminate effect on the money supply.
2/ When a bank loans out $1,000, the money supply
a. does not change.
b. decreases.
c. increases.
d. may do any of the above.
3/ If the reserve ratio is 5 percent and a bank receives a new deposit of
$500, this bank
a. must increase its total reserves by $25.
b. will initially see its total reserves increase by $500.
c. will be able to make a new loan of $475.
d. All of the above are true.
Use the balance sheet below for question 4
Chapter 29: The Monetary System
92
Last Bank of Cedar Bend
Assets
Reserves: $25,000
Liabilities
Deposits: $150,000
Loans: $125,000
4/ Refer to the balance sheet above. If the reserve requirement is 10 percent
and then someone deposits $50,000 into the bank, it will
a. have $65,000 in excess reserves.
b. have $55,000 in excess reserves.
c. need to raise reserves by $5,000.
d. None of the above are correct.
5/ If a bank uses $80 of reserves to make a new loan when the reserve ratio
is 25 percent,
a. the money supply initially decreases by $80.
b. the money supply initially increases by $20.
c. the money supply will eventually increase by more than $20 but less
than $80.
Chapter 29: The Monetary System
93
d. the level of wealth in the economy will not change.
6/ In Wellville, the money supply is $80,000 and reserves are $18,000.
Assuming that people hold only deposits and no currency, and that banks
hold only required reserves, the required reserve ratio is
a. 29 percent.
b. 22.5 percent.
c. 16 percent.
d. None of the above are correct.
7/ If the reserve ratio is 100 percent, depositing $500 of paper money in a
bank will eventually increase the money supply by
a. $5,000.
b. $1,000.
c. $500.
d. $0.
8/If reserve requirements are increased, the reserve ratio
a. increases, the money multiplier increases, and the money supply
increases.
Chapter 29: The Monetary System
94
b. increases, the money multiplier decreases, and the money supply
decreases.
c. decreases, the money multiplier increases, and the money supply
increases.
d. decreases, the money multiplier decreases, and the money supply
increases.
9/If the reserve ratio is 20 percent, and banks do not hold excess reserves,
when the Fed sells $40 million of bonds to the public, bank reserves
a. increase by $40 million and the money supply eventually increases by
$200 million.
b. increase by $40 million and the money supply eventually increases by
$800 million.
c. decrease by $40 million and the money supply eventually decreases by
$200 million.
d. decrease by $40 million and the money supply eventually decreases by
$800 million.
10/The banking system has $10 million in reserves, the reserve requirement
is 20 percent, and there are no excess reserves. The public holds $10 million
in cash. Then bankers decide that it is prudent to hold some excess
Chapter 29: The Monetary System
95
reserves, and so begin to hold 25 percent of deposits in the form of reserves.
Other things the same, this action will cause the money supply to
a. change forms, but not size.
b. fall by $10 million.
c. fall by $5 million.
d. fall by $.5 million
11. The Fed can increase the price level by conducting open market
a. Sales, raising the discount rate, and lowering requirement ratio
b. sales, lowering the discount rate, and increasing requirement ratio
c. purchases, raising the discount rate, and increasing requirement ratio
d. purchases, lowering the discount rate, and lowering requirement ratio
12. At one time, the country of Gia Lai had no banks, but had currency of
$10 million. Then a banking system was established with a reserve
requirement of 20 percent. The people of Aquilonia deposited half of their
currency into the banking system. If banks do not hold excess reserves,
what is Gia Lai’s money supply now?
a. $10 million
b. $12 million
Chapter 29: The Monetary System
c. $25 million
d. $30 million
13. Money has:
A. 3 functions: Medium of Exchange, Unit of Account, Store of Value
B. 1 function: Buying goods
C. 2 functions: Store of Value, Medium of Exchange
D. 3 functions like Answer A but only with Fiat Money
14. Choose the correct answer:
(i) Fiat Money has intrinsic value
(ii) There are 2 types of money: Fiat Money and Commodity Money
(iii) Gold and Cigars are examples of Fiat Money
(iv) Dollars, VietNamDong are examples of Fiat Money
(v) Currency refers to the money in the hand of public
(vi) Demand Deposit refer to the balances in the bank account
(vii) In the T-account, Asset can be different from Liabilities
A. (i), (iii), (vii)
B. (ii), (iv), (v), (vi)
96
Chapter 29: The Monetary System
97
C. All is correct
D. None is correct
Table 2:
Asset
Liabilities
Required Reserve: $200
Deposit: $1500
Excess Reserve: $100
Debt: $350
Loans: $1300
Owner’s Equity: ?
Security: $400
15. Refer to Table 2. What is Owner’s Equity:
A. $150
B. $1500
C. $400
D. There is not enough information given
16. Refer to Table 2. What is Reserve Ratio and Requirement Reserve,
respectively:
Chapter 29: The Monetary System
98
A. 10% and 15%
B. 15% and 10%
C. 20% and 13.3%
D. Cannot be calculated since Owner’s Equity is missing
ANSWER AND EXPLANATION
1. A (Explain: When someone deposit money into a demand deposit, money
supply do not change (Theoretical Information))
2. C (Explain: When the bank loan out money, the depositors still have demand
deposits, but now the borrowers hold more currency, so money supply
increases. In other words, as a bank creates the asset of money, it also creates a
corresponding liability for those who borrowed the created money, but the
economy is not wealthier.)
3. D (Explain:
A is correct. The reserve ratio is 5%, so the Reserves must equal 5% of the new
deposit. 5% * $500 = $25
B is correct. The depositor deposits $500, so, initially, the bank reserves will
increase by $500
Chapter 29: The Monetary System
99
C is correct. The reserve ratio is 5%, so the bank can do whatever it wants with
the remaining 95%, it can lend out or put it in Excess Reserves. In this case, the
bank is able to make new loan at the maximum amount of $475)
4. B
(Explain: When someone deposits $50,000 into the bank, the bank now have
$200,000 in total deposits. The reserve requirement is 10%, so the Required
Reserve is $20,000 => Excess Reserves now equals $5,000. The bank now have
$200,000 (Deposits) - $25,000 (Reserves) - $125,000 (Loans) = $50,000
remaining. Since the bank doesn’t lend out more loan, the $50,000 will be
added to Excess Reserves.
So now Excess Reserves totals $55,000)
5.D (Explain: When the bank uses $80 to make new loan. The money supply
will initially increase by $80. Eventually, the money supply will increase by
$320. When the bank lend out money and create more money supply, the level
of wealth in the economy will not change)
6. B (Explain: The bank holds only Require Reserves, so Reserves = Required
Reserves = $18,000. The Required Reserve Ratio
= $18000
× 100%=
$80000
22.5%. Note that Reserves is included in the money supply).
Chapter 29: The Monetary System
100
7.D (Explain: When you deposit $500 of currency into demand deposit, money
supply will not change. And since the reserve ratio of the bank is 100%, it does
not lend out money, hence cannot increase money supply).
8.B (Explain:
Reserve Requirements are the minimum of Reserve Ratio, so when Reserve
Requirements increase, Reserve Ratio increases
The Money Multiplier
=
1
𝑅𝑒𝑠𝑒𝑟𝑣𝑒 𝑅𝑎𝑡𝑖𝑜
, so when Reserve Ratio increases,
Money Multiplier decreases
Money Multiplier is the determinant of the magnitude of Money Supply change,
so when Money Multiplier decreases, Money Supply will decrease)
9.C (Explain: The Fed is currently performing Open market Sales, which will
decrease Money Supply.
Initially, the bank reserves fall by $40 million due to the sales of bonds
The Money Multiplier
=
1
𝑅𝑒𝑠𝑒𝑟𝑣𝑒 𝑅𝑎𝑡𝑖𝑜
=5
So eventually the Money Supply will decrease by $40 million * 5 = $200
million)
10.B (Explain:
Chapter 29: The Monetary System
101
As the reserve ratio have increased from 20% to 25%, the money supply falls.
At the rate of 20%, Reserves = $10M => Deposits = $50M
At the rate of 25%, New Reserves = $12.5M
As the reserve ratio increased from 20% to 25%, the reserves increased $2.5M.
So the bank has $2.5 million less to lend out, which decrease money supply by
$10M)
11.D (Explain:
For Price Level to increase, Money Supply must increase
When the Fed perform open market purchase, the Fed is buying bonds from the
banks, the money will go to the bank, thus the bank now has more money to
lends out => Money Supply increases
Discount rate is used when the Fed lends money to bank. When discount rate
decreases, the bank will be encouraged to borrow money from the Fed, so
money supply will increase.
When the Reserve Requirement Ratio decreases, the amount the bank needs to
reserve is smaller, the amount the bank can lend out is larger => Money supply
increases)
12.D (Explain: Money supply from the banks:
Chapter 29: The Monetary System
102
When people deposits $5M in the bank, the bank creates $25M of money supply
Money supply from currency held by citizens = $10M - $5M = $5M
Total money supply: $25M + $5M = $30M.)
13.A (Explain: Theoretical Information)
14. B(Explain:
(i): Wrong. Fiat Money doesn’t have intrinsic value, but Commodity Money has
(ii): Correct. Theoretical Information
(iii): Wrong. Gold and Cigars are examples of Commodity Money
(iv): Correct. National Currency is the example of Fiat Money
(v), (vi): Correct. Money can be in the hand of public (Currency) or in balances
in the bank account (Demand Deposit)
(vii): Wrong. In the T-account, Asset must equal Liabilities
15.A (Explain: Asset = Liabilities)
16.C (Explain:
Reserve Ratio =
𝑅𝑒𝑞𝑢𝑖𝑟𝑒𝑑 𝑅𝑒𝑠𝑒𝑟𝑣𝑒 + 𝐸𝑥𝑐𝑒𝑠𝑠 𝑅𝑒𝑠𝑒𝑟𝑣𝑒
Requirement Reserve =
𝐷𝑒𝑝𝑜𝑠𝑖𝑡
𝑅𝑒𝑞𝑢𝑖𝑟𝑒𝑑 𝑅𝑒𝑠𝑒𝑟𝑣𝑒
𝐷𝑒𝑝𝑜𝑠𝑖𝑡
x 100%)
x 100%
Chapter 30: Money Growth and Inflation
103
Chapter 30
Money Growth and Inflation
I- The Classical Theory of Inflation:
1. The Level of Prices and the Value of Money:
- Inflation: economy-wide phenomenon that concern the value of the
economy’s medium of exchange.
- The Price Level (P) measured by CPI or GDP Deflator is the number of
dollars needed to buy a basket of goods and services.
- The Value of Money (1/P): the quantity of goods and services that can be
bought with $1.
● Price level increases => Money value decreases
2. Money Supply, Money Demand and Money Equilibrium:
- Money supply is determined by the Fed.
- Money demand reflects how much wealth people want to hold in liquid
form.
- The Quantity of Money demanded depends on:
+ The interest rate that a person could earn by using the money to
buy interest-bearing bond rather than just leaving it.
Chapter 30: Money Growth and Inflation
104
+ The Price level: P level increases => Q demanded increases.
3. The Effects of a Monetary Injection:
- Monetary injection: shifts supply curve to the right
Chapter 30: Money Growth and Inflation
105
- Quantity theory of money:
+ The quantity of money determines Price level
+ The growth rate in the quantity of money available determines the
inflation rate.
4. A Brief Look at the Adjustment Process:
Monetary Injection: Create excess Supply of Money -> increase Demand for
goods and services -> Increase in the Price -> Increase in the quantity of money
demanded -> Economy reach new equilibrium (where 𝑄𝑀𝑆 =𝑄𝑀𝐷 ).
Chapter 30: Money Growth and Inflation
106
5. The Classical Dichotomy and Monetary Neutrality:
- Classical Dichotomy: The theoretical separation of real and nominal
variables.
+ Nominal: measured in monetary unit - Dollar price
+ Real: physical unit, relative Price, real wage, real interest rate,
production, employment.
- Monetary Neutrality: the proposition that changes in the money supply do
no affect real variables.
=> True in the Long-Run, might not be true in the Short-Run
Chapter 30: Money Growth and Inflation
107
6. Velocity and the Quantity Equation:
- Velocity of Money: the speed at which money change hand.
MxV=PxY
With
M: the Quantity of Money
V: Velocity of Money
P: the Price Level (the GDP Deflator)
Y: real GDP
- For (P x Y) of spending to take place with only M of money, each dollar
bill must change hand average V times.
- V ≈ constant. When M increase => P increase => Inflation.
7. The Inflation Tax:
- Government prints money as a way to gain revenue-> P increases.
- Inflation tax: tax on everyone who holds money
8. The Fisher Effect:
Chapter 30: Money Growth and Inflation
108
Real interest rate = Nominal interest rate - Inflation rate
Fisher effect: Money Supply increase -> Nominal interest rate increases,
inflation increase (LR result).
II- The Cost of Inflation:
1. The Inflation Fallacy:
- Inflation Fallacy: Decrease in Purchasing Power
P increase -> pay more -> receive more -> inflation does not itself reduce
people’s purchasing power.
2. Shoeleather Costs:
- Inflation tax can be avoided by holding less money
- Shoeleather cost -> more trip to the bank
3. Menu Costs:
- Menu cost: cost of changing P
- High inflation -> change P -> waste money in printing new menu.
4. Relative - Price Variability and the Misallocation of Resources:
Chapter 30: Money Growth and Inflation
109
Inflation causes relative price to vary more than they otherwise would ->
misallocation.
5. Inflation-Induced Tax Distortions:
- Tax Distortion: Lawmaker often fail to take inflation into account.
Inflation raise the tax burden.
- Tax treatment on capital gain: increase size of capital gain -> increase tax
burden
- Income tax treats the nominal interest rate.
6. Confusion and Inconvenience:
- Inflation cause value of dollar to be different, computing profit
- Less able to sort successful and unsuccessful firms
7. A Special Cost of Unexpected Inflation: Arbitrary Redistributions of
Wealth:
- Arbitrary redistribution of wealth: special cost of unexpected inflation.
- Unexpected change -> redistribute wealth among creditors and debtors.
+ Low average inflation -> stable.
+ High average inflation -> unstable.
8. Inflation is Bad but Deflation May Be Worse:
Chapter 30: Money Growth and Inflation
- Small, predictable amount of deflation is desirable
- Cost: Menu cost, relative-p variability, redistribution of wealth
EXERCISE
1. Inflation can be measured by the
a. change in the consumer price index.
b. percentage change in the consumer price index.
c. percentage change in the price of a specific commodity.
d. change in the price of a specific commodity.
2. The supply of money is determined by
a. the price level.
b. the Treasury and Congressional Budget Office.
c. the Federal Reserve System.
d. the demand for money.
110
Chapter 30: Money Growth and Inflation
111
3. When the money market is drawn with the value of money on the
vertical axis, the price level increases if
a. either money demand or money supply shifts right.
b. either money demand or money supply shifts left.
c. money demand shifts right or money supply shifts left.
d. money demand shifts left or money supply shifts right.
4. When the money market is drawn with the value of money on the
vertical axis, an increase in the price level causes a
a. shift to the right of the money demand curve.
b. shift to the left of the money demand curve.
c. movement to the left along the money demand curve.
d. movement to the right along the money demand curve.
5. As the price level decreases, the value of money
a. increases, so people want to hold more of it.
b. increases, so people want to hold less of it.
Chapter 30: Money Growth and Inflation
112
c. decreases, so people want to hold more of it.
d. decreases, so people want to hold less of it.
6. When the money market is drawn with the value of money on the
vertical axis, if the price level is higher the equilibrium price level, there is
a
a. shortage, so the price level will rise.
b. shortage, so the price level will fall.
c. surplus, so the price level will rise.
d. surplus, so the price level will fall.
Figure 1
Chapter 30: Money Growth and Inflation
113
7. Refer to Figure 1.
When the money supply curve shifts from MS1 to MS2,
a. the demand for goods and services decreases.
b. the economy's ability to produce goods and services increases.
c. the equilibrium price level increases.
d. the equilibrium value of money increases.
8. The price of a Honda Accord divided by the price of a Honda Civic is a
a. classical variable.
b. dichotomous variable.
c. nominal variable.
d. real variable.
9. According to the classical dichotomy, when the money supply doubles,
which of the following also double?
a. the price level and nominal wages
b. the price level, but not the nominal wage
Chapter 30: Money Growth and Inflation
114
c. the nominal wage, but not the price level
d. neither the nominal wage nor the price level
10. Monetary neutrality implies that an increase in the quantity of money
will
a. increase employment.
b. increase the price level.
c. increase production.
d. not affect the price level.
11. Velocity is computed as
a. (P x Y)/M.
b. (P x M)/Y.
c. (Y x M)/P.
d. (Y x M)/V.
Chapter 30: Money Growth and Inflation
115
12. Assuming that V is constant, the quantity equation implies that an
increase in M could result in
a. an increase in the price level.
b. an increase in real GDP.
c. an increase in nominal GDP.
d. any of the above.
13. Last year Tealandia produced 50,000 bags of green tea, which sold at 4
units each of Tealandia’s currency—the Leaf. Tealandia’s money supply
was 10,000. What was the velocity of money in Tealandia?
a. 20
b. 5
c. 1/20
d. 1/5
14. The money supply in Freedonia is $200 billion. Nominal GDP is $800
billion and real GDP is $400 billion. Assuming that velocity is stable, if real
GDP grows by 10 percent this year, and if the money supply does not
change this year, then the
a. inflation rate will be zero.
Chapter 30: Money Growth and Inflation
116
b. price level will fall by 9.09 percent.
c. price level will rise by 10 percent.
d. None of the above is correct.
15. Velocity in the country of Shem is always stable. In 2002, the money
supply was $200 billion and the GDP price deflator was four times as high
as it was in the base year. In 2003, the money supply increased to $240
billion, the price level increased by 15 percent, and nominal GDP equaled
$1,200 billion. By how much did real GDP increase between 2002 and
2003?
a. 20 percent
b. 4.35 percent
c. 2.17 percent
d. There is not enough information to answer the question.
16. Printing money to finance government expenditures
a. causes the value of money to rise.
b. imposes a tax on everyone who holds money.
Chapter 30: Money Growth and Inflation
117
c. is the principle method by which the U.S. government finances its
expenditures.
d. None of the above is correct.
17. Greta puts money in a savings account at her bank earning 4.5 percent.
One year later she takes her money out and notes that while her money was
earning interest, prices rose 2.5 percent. Greta now has
a. 4.5 percent more money with which she can purchase 7 percent
more goods.
b. 4.5 percent more money with which she can purchase 2 percent
more goods.
c. 7 percent more money with which she can purchase 7 percent more
goods.
d. 7 percent more money with which she can purchase 2 percent
fewer goods.
18. If a country had deflation,
a. the nominal interest rate would be greater than the real interest rate.
b. the real interest rate would be greater than the nominal interest rate.
Chapter 30: Money Growth and Inflation
118
c. the real interest rate would equal the nominal interest rate.
d. None of the above are necessarily correct.
19. Shoe-leather costs refer to
a. the cost of more frequent price changes induced by higher
inflation.
b. the distortion in resource allocation created by distortions in
relative prices due to inflation.
c. resources used to maintain lower money holdings when inflation is
high.
d. the distortion in incentives created by inflation by taxes that do not
adjust for inflation.
20. You put money in an account and earn a real interest rate of 4 percent.
Inflation is 2 percent, and your marginal tax rate is 20 percent. What is
your after-tax real rate of interest?
a. 1.2 percent
b. 2.8 percent
Chapter 30: Money Growth and Inflation
119
c. 4.8 percent
d. None of the above is correct.
21. If inflation is more than expected,
a. creditors receive a lower real interest rate than they had anticipated.
b. creditors pay a lower real interest rate than they had anticipated.
c. debtors receive a higher real interest rate than they had anticipated.
d. debtors pay a higher real interest rate than they had anticipated.
ANSWER AND EXPLANATION
1. ANSWER: b. percentage change in the consumer price index.
(Explain:
Inflation in year 2 =
𝐶𝑃𝐼 𝑦𝑒𝑎𝑟 2 − 𝐶𝑃𝐼 𝑦𝑒𝑎𝑟 1
𝐶𝑃𝐼 𝑦𝑒𝑎𝑟 1
× 100%
So Inflation rate of a year is the percentage change in CPI in that year.
Inflation in year 2 =
100%
𝐺𝐷𝑃 𝐷𝑒𝑓𝑙𝑎𝑡𝑜𝑟 𝑦𝑒𝑎𝑟2 − 𝐺𝐷𝑃 𝐷𝑒𝑓𝑙𝑎𝑡𝑜𝑟 𝑦𝑒𝑎𝑟1
𝐺𝐷𝑃 𝐷𝑒𝑓𝑙𝑎𝑡𝑜𝑟 𝑦𝑒𝑎𝑟1
×
Chapter 30: Money Growth and Inflation
120
Inflation rate of a year is also the percentage change in GDP Deflator in
that year (or the percentage in price level of all goods included in GDP in that
year))
2. ANSWER: c. the Federal Reserve System.
(Explain: Theoretical Information. Money supply of an economy is controlled
by the Federal Reserve System (The Fed))
3. ANSWER: d. money demand shifts left or money supply shifts right.
(Explain: In order for Price Level to increase, the equilibrium point A must
move downward. In order for point A to move downward, Money Demand
curve must shift left or Money Supply curve must shift right
)
4. ANSWER: d. movement to the right along the money demand curve.
Chapter 30: Money Growth and Inflation
121
(Explain: An increase in Price Level cause the equilibrium point A move
downward along the Money Demand curve.
)
(*Note: When working with a graph, a change in variables shown in vertical
axis (axes) will only cause movement along the curve. Other variables will
either shift the curves or have no effect.
E.g.: In Question 4, Price Level and Value of Money are variables shown
in the vertical axes, so a change in Price Level and Value of Money will only
cause movement along the curve. )
5. ANSWER: b. increases, so people want to hold less of it.
Chapter 30: Money Growth and Inflation
122
1
(Explain: When the Price level (P) decreases, the Value of Money ( ) increases.
𝑃
When the Value of Money increases, people need less money to perform the
same transactions as before, so they want to hold less money.
We can also put the situation in a graph, when the Value of Money
increases, the equilibrium point A must move upward => Quantity of Money
Demanded is lower.
)
6.ANSWER: b. shortage, so the price level will fall.
(Explain: When the Price Level is larger than the equilibrium Price Level, there
is a shortage. Eventually, the money market will shift back to equilibrium, so
Price Level will fall back to equilibrium.
Chapter 30: Money Growth and Inflation
123
)
7. ANSWER: c. the equilibrium price level increases.
(Explain:
When MS1 shifts to MS2, the equilibrium shifts from point A to point D,
Price Level (P) increases.)
8. ANSWER: d. real variable.
(Explain: Recall the formula of Relative Price:
Relative Price of Good A compared to good B =
𝑃𝑟𝑖𝑐𝑒 𝑔𝑜𝑜𝑑 𝐴
𝑃𝑟𝑖𝑐𝑒 𝑔𝑜𝑜𝑑 𝐵
Relative price is a real variable because it is measured in terms of
physical units)
9. ANSWER: a. the price level and nominal wages
Chapter 30: Money Growth and Inflation
124
(Explain: According to classical dichotomy, money supply only affects nominal
variable.
Price level and nominal wage are nominal variables because they are
measured in terms of monetary units.
When money supply increases, Money Supply curve shifts to the right,
resulting in an increase in Price Level.
When money supply increases, inflation rate increases, resulting the
decrease of the value of each dollars. In order for the nominal wage to have the
same purchasing power as before, nominal wage must increase.
)
10. ANSWER: b. increase the price level.
Chapter 30: Money Growth and Inflation
125
(Explain: Money Neutrality suggests that change in quantity of money (Money
Supply) only affects nominal variables, not real variables.)
11. ANSWER: a. (P x Y)/M.
(Explain: Theoretical Information)
12. ANSWER: d. any of the above.
(Explain: M . V = P . Y
When V is constant and M increases, either P or Y must increase. This increase
could be resulted from:
+ Increase in Price Level (P)
+ Increase in Quantity of Output (Y)
+ Increase in both of the above
13. ANSWER: a. 20
(Explain: M = Money Supply = 10,000
P = Price Level = 4 Tealandia’s currency
Y = Quantity of Output = 50,000 units
Use the formula M . V = P . Y)
14. ANSWER: b. price level will fall by 9.09 percent.
Chapter 30: Money Growth and Inflation
126
(Explain: P can be GDP Deflator
Y can be Real GDP
Money Supply (M) and Velocity of Money (V) does not change after the
Real GDP increases => GDP Deflator must decrease
=> P1 . Y1 = P2 . Q2
⇔ P1 . Y1 = P1 . ( 1 - a) . Y1 . ( 1 + b)
Where: a: percentage decreased in GDP Deflator
B: percentage increased in Real GDP
⇔ 1 = ( 1- a) . (1+b)
⇔ 1 = (1-a) . (1 + 10%)
⇔ a = 9.09%)
15. ANSWER: b. 4.35 percent
(Explain:
GDP Deflator in the base year = 100
=> GDP Deflator in 2002 = 400
Real GDP =
𝑁𝑜𝑚𝑖𝑛𝑎𝑙 𝐺𝐷𝑃
𝐺𝐷𝑃 𝐷𝑒𝑓𝑙𝑎𝑡𝑜𝑟
Since V is constant
=> Y2 =
60
23
Chapter 30: Money Growth and Inflation
127
V1 = V2
⇔ Y1 = 2.5
=> Real GDP increases by 4.35%)
16. ANSWER: b. imposes a tax on everyone who holds money.
(Explain: A is wrong. Printing money cause Money Supply to increase =>
Value of Money decreases
B is correct. Printing money creates Inflation => causing situation like a
tax (Inflation Tax) because Inflation erodes the value of the money in your
wallet.
C is wrong. The principle method the US government use to finance its
expenditures is through levying tax.)
17. ANSWER: b. 4.5 percent more money with which she can purchase 2
percent more goods.
(Explain:
She earns 4.5% interest, which means after a year, she will have 4.5%
more money.
Real Interest = Nominal Interest + Inflation Rate
=> Real Interest = 2%
Chapter 30: Money Growth and Inflation
128
She will have 4.5% more money but only have 2% more purchasing
power because of the inflation)
18. ANSWER: b. the real interest rate would be greater than the nominal
interest rate.
(Explain: Deflation occurs when Inflation Rate is negative
Inflation Rate = Real Interest Rate - Nominal Interest Rate
=> In order for Deflation to occur, Real Interest must be greater than
Nominal Interest)
19. ANSWER: c. resources used to maintain lower money holdings when
inflation is high.
(Explain: A is wrong. A is the effect of Menu Cost
B is wrong. B is the effect of Relative Price Variability and Misallocation
of Resources
C is correct. C is the effect of Shoe-leather Cost. Inflation erodes the
value of the money in your wallet => People will keep more money in the bank,
withdraw less money, and make more trips to the bank. Shoe-leather Cost refers
to the time and resources used to make more trips to the bank.
D is wrong. D is the effect of Tax Distortion)
Chapter 30: Money Growth and Inflation
129
20. ANSWER: b. 2.8 percent
(Explain: Tax only affects Nominal Interest rate.
Compute the new Nominal Interest rate affected by tax, then compute the
new Real Interest rate after tax (= 2.8%))
21. ANSWER: a. creditors receive a lower real interest rate than they had
anticipated.
(Explain: When the loans are made, creditors and debtors set up Nominal
Interest Rate based on Expected Inflation Rate.
When Inflation Rate is more than expected, debtors now have to pay back
creditors less real value than before (because Inflation erodes value of money
and Nominal Rate is unchanged) and vice versa.
=> Creditors receive a lower real interest rate than they had anticipated.)
Chapter 31: Open-Economy Macroeconomics: Basic Concepts
Chapter 31
Open-Economy Macroeconomics: Basic Concepts
I/ Exports, Imports, and Net Exports:
Trade balance = Net Export = Exports - Imports
● E > I: trade surplus
● E < I: trade deficit
● E = I: balanced trade
Factors affecting NX:
- Tastes of consumers for domestic and foreign goods
- Prices of domestic and foreign goods
- Exchange rates between domestic currency and foreign currency
- Incomes of consumers at home and abroad
- Cost of transporting goods
- Government policies towards trading
130
Chapter 31: Open-Economy Macroeconomics: Basic Concepts
131
II/ The flow of financial resources: Net Capital Outflow:
Net Capital Outflow = Net Foreign Investment
= Purchase of foreign assets by domestic residents - Purchase of domestic assets
by foreigners
NCO can be positive or negative:
+ When NCO > 0, the country is said to experience Capital Outflow
+ When NCO < 0, the country is said to experience Capital Inflow
There are 2 forms of NCO:
● Foreign Direct Investment: The owner actively manages the
investment.
E.g.: McDonald opens restaurants in Asia.
● Foreign Portfolio Investment: The owner is more passive at managing
the investment.
E.g.: The USA buys stocks in Russia.
Factors affecting NCO:
- Real interest rate paid on foreign assets
Chapter 31: Open-Economy Macroeconomics: Basic Concepts
132
- Real interest rate paid on domestic assets
- Perceived economic and political risks of holding assets abroad
- Government policies that affect foreign ownership of domestic assets
III/ The equality of NX and NCO:
NCO = NX
Explain:
When America exports (NX increases), other countries will pay America
foreign currency instead of USD. With foreign currency, America will buy
foreign assets (NCO increases) and vice versa.
IV/ Savings, Investment and their relationship to the
International Flows:
For an open economy:
Saving = Domestic Investment + NCO
Chapter 31: Open-Economy Macroeconomics: Basic Concepts
133
Whenever a US citizen saves a dollar (Saving increases), he/she will use
it to buy domestic assets (Domestic Investment increases ) or foreign assets (
NCO increases)
V/ The price for international transaction:
Nominal exchange rate: the rate at which a person can trade the currency of
one country for the currency of another.
E.g.: Exchange rate = 80 yen/ dollar => 1 dollar can be exchanged for 80 yen
and vice versa.
Appreciation: an increase in the value of a currency as measured by the amount
of foreign currency it can buy
E.g.: 1 USD = 22,000 VND
→
1 USD = 23,000 VND
=> We say that the dollars appreciate (or strengthen)
Depreciation: a decrease in the value of a currency as measured by the amount
of foreign currency it can buy
E.g.: 1 Euro = 25,000 VND
→
1 Euro = 22,000 VND
Chapter 31: Open-Economy Macroeconomics: Basic Concepts
134
=> We say that the Euros depreciate (or weaken)
Real exchange rate: the rate at which a person can trade the goods and services
of one country for the goods and services of another
Real exchange rate =
𝑁𝑜𝑚𝑖𝑛𝑎𝑙 𝑒𝑥𝑐ℎ𝑎𝑛𝑔𝑒 𝑟𝑎𝑡𝑒 × 𝐷𝑜𝑚𝑒𝑠𝑡𝑖𝑐 𝑝𝑟𝑖𝑐𝑒
𝐹𝑜𝑟𝑒𝑖𝑔𝑛 𝑝𝑟𝑖𝑐𝑒
Rule of variables:
Real exchange rate
=
𝑁𝑜𝑚𝑖𝑛𝑎𝑙 𝑒𝑥𝑐ℎ𝑎𝑛𝑔𝑒 𝑟𝑎𝑡𝑒 (𝐶𝑢𝑟𝑟𝑒𝑛𝑐𝑦 1/ 𝐶𝑢𝑟𝑟𝑒𝑛𝑐𝑦 2) × 𝐷𝑜𝑚𝑒𝑠𝑡𝑖𝑐 𝑝𝑟𝑖𝑐𝑒 (𝐶𝑢𝑟𝑟𝑒𝑛𝑐𝑦 1)
𝐹𝑜𝑟𝑒𝑖𝑔𝑛 𝑝𝑟𝑖𝑐𝑒 (𝐶𝑢𝑟𝑟𝑒𝑛𝑐𝑦 2)
= … Currency 1 per Currency 2
E.g.:
Real exchange rate
=
𝑁𝑜𝑚𝑖𝑛𝑎𝑙 𝑒𝑥𝑐ℎ𝑎𝑛𝑔𝑒 𝑟𝑎𝑡𝑒 (𝑈𝑆𝐷/𝑉𝑁𝐷) × 𝐷𝑜𝑚𝑒𝑠𝑡𝑖𝑐 𝑝𝑟𝑖𝑐𝑒 (𝑉𝑁𝐷)
= … USD per VND
𝐹𝑜𝑟𝑒𝑖𝑔𝑛 𝑝𝑟𝑖𝑐𝑒 (𝑈𝑆𝐷)
Chapter 31: Open-Economy Macroeconomics: Basic Concepts
135
Real exchange rate is the key determinant of how much to export and import
Overall real exchange rate between the US and other country:
Real exchange rate =
𝑁𝑜𝑚𝑖𝑛𝑎𝑙 𝐸𝑥𝑐ℎ𝑎𝑛𝑔𝑒 𝑅𝑎𝑡𝑒 𝑏𝑒𝑡𝑤𝑒𝑒𝑛 𝑈𝑆𝐷 𝑎𝑛𝑑 𝑓𝑜𝑟𝑒𝑖𝑔𝑛 𝑐𝑢𝑟𝑟𝑒𝑛𝑐𝑦 ×𝑈𝑆 𝑝𝑟𝑖𝑐𝑒 𝑖𝑛𝑑𝑒𝑥
𝑂𝑡ℎ𝑒𝑟 𝑐𝑜𝑢𝑛𝑡𝑟𝑦′𝑠 𝑝𝑟𝑖𝑐𝑒 𝑖𝑛𝑑𝑒𝑥
Real exchange rate ↘ => US goods are cheaper relative to other countries’
goods => Export ↗ ; Import ↘ => NX ↗
Real exchange rate ↗ => US goods are more expensive relative to other
countries’ goods => Export ↘ ; Import ↗ => NX ↘
VI/ Purchasing - Power Parity:
A theory of exchange rates whereby a unit of any given currency should
be able to buy the same quantity of goods in all countries (same real value in all
countries).
Based on law of one price: If an identical good has lower price in place
A than place B. People will buy goods from A and sell them in B for profit
Chapter 31: Open-Economy Macroeconomics: Basic Concepts
136
(arbitrage). Eventually, this will drive up demand in place A and supply in place
B => Increase price in place A and decrease price and place B until prices on
both places are equal.
Purchasing-power parity implies that Real exchange rate between 2 countries
equals 1:
Real Exchange rate =
𝑁𝑜𝑚𝑖𝑛𝑎𝑙 𝐸𝑥𝑐ℎ𝑎𝑛𝑔𝑒 𝑟𝑎𝑡𝑒 × 𝑈𝑆 𝑝𝑟𝑖𝑐𝑒 𝑖𝑛𝑑𝑒𝑥
𝑂𝑡ℎ𝑒𝑟 𝑐𝑜𝑢𝑛𝑡𝑟𝑖𝑒𝑠′ 𝑝𝑟𝑖𝑐𝑒 𝑖𝑛𝑑𝑒𝑥
=1
According to Purchasing-power parity, Nominal exchange rate between
the currencies of 2 countries must reflect the price level of that 2 countries.
Money printed ↗ => money value ↘ in terms of goods and services and
currencies it can buy
Limitations of purchasing-power parity:
Purchasing-power parity is not completely accurate, do not ensure dollars
has the same value in all countries because:
- Not every goods are easily traded with each others
Chapter 31: Open-Economy Macroeconomics: Basic Concepts
137
- Not every good is a perfect substitute of others
These two reasons stop Arbitrage from functioning effectively, thus the
difference in prices in two places is hard to eliminate.
=> In reality, Real exchange rate fluctuates.
EXERCISE
1. Oceania buys $100 of wine from Escudia and Escudia buys $40 of
wool from Oceania. What are the net exports of Oceania and Escudia
in that order?
a. $140 and $140
b. $100 and $40
c. $60 and -$60
d. None of the above is correct.
2. In 2001, Denmark had net exports of $8.3 billion and sold $52.4
billion of goods and services abroad. Denmark had
a. $60.7 billion of imports, $52.4 billion of imports, and a
balanced trade.
Chapter 31: Open-Economy Macroeconomics: Basic Concepts
138
b. $60.7 billion of exports, $52.4 of imports, and a balanced trade
c. $52.4 billion of imports, $44.1 billion of exports, and a trade
deficit
d. $52.4 billion of exports, $44.1 billion of imports, and a trade
surplus
3. Trade balance is different from balanced trade is that:
A. Trade balance is import, balanced trade is a synonym of trade
surplus
B. Trade balance is export, balanced trade is a synonym of trade
deficit
C. Trade balance is Net Capital Outflow, balanced trade is the
negative value of Net Export
D. Trade balance is Net Export, balanced trade is when Net Export =
0.
4. Larry, a U.S. citizen, opens and operates a bookstore in Spain. This
counts as
a. investment for Larry and U.S. foreign direct investment.
b. investment for Larry and U.S. foreign portfolio investment.
Chapter 31: Open-Economy Macroeconomics: Basic Concepts
139
c. U.S. foreign direct investment and U.S. domestic investment.
d. U.S. foreign portfolio investment and U.S. domestic investment.
5. Bolivia buys railroad engines from a U.S. firm and pays for them
with Bolivianos (Bolivian currency). By itself, this transaction
a. increases both U.S. net exports and U.S. net capital outflow.
b. decreases both U.S. net exports and U.S. net capital outflow.
c. increases U.S. net exports and does not affect U.S. net capital
outflow.
d. None of the above is correct.
6. Net capital outflow refers to the purchase of
a. foreign assets by domestic residents minus the purchase of
domestic assets by foreign residents.
b. foreign assets by domestic residents minus the purchase of
foreign goods and services by domestic residents.
c. domestic assets by foreign residents minus the purchase of
domestic goods and services by foreign residents.
Chapter 31: Open-Economy Macroeconomics: Basic Concepts
140
d. domestic assets by foreign residents minus the purchase of
foreign assets by domestic residents.
7. Choose the correct answer:
(i) KFC opens its restaurants in India
(ii) A US citizen buys 10 shares of stock in Trung Nguyên
(iii) Disney builds a new amusement park in Paris
(iv) Đào operates a bánh tráng trộn stall in the US
(v) Nolva in the US buys bonds issued by a Turkish company
Which of the above is Foreign Direct Investment?
A. (i), (iii), (iv)
B. (ii), (v)
C. All of the above
D. None of the above
8. A country has $100 million of net exports and $170 million of saving.
Net capital outflow is
a. $70 million and domestic investment is $170 million.
b. $70 million and domestic investment is $270 million.
Chapter 31: Open-Economy Macroeconomics: Basic Concepts
141
c. $100 million and domestic investment is $70 million.
d. None of the above is correct.
9. If a country has an increase in quality of goods that are relatively
attractive to other countries, we would expect it to have
a. both positive net exports and positive net capital outflow.
b. both negative net exports and negative net capital outflow.
c. positive net exports and negative net capital outflow.
d. negative net exports and positive net capital outflow.
10. Suppose the real exchange rate is 1/2 gallon of Canadian gasoline per
gallon of U.S. gasoline, a gallon of U.S. gasoline costs $1.50 U.S., and
a gallon of Canadian gas costs $3.90 Canadian. What is the nominal
exchange rate?
a. .385 Canadian dollars per U.S. dollar
b. .65 Canadian dollars per U.S. dollar
c. 1.30 Canadian dollars per U.S. dollar
d. None of the above is correct.
Chapter 31: Open-Economy Macroeconomics: Basic Concepts
142
11. A U.S. firm buys sardines from Morocco and pays for them with U.S.
dollars. Other things the same, U.S. net exports
a. increase, and U.S. net capital outflow increases.
b. increase, and U.S. net capital outflow decreases.
c. decrease, and U.S. net capital outflow increases.
d. decrease, and U.S. net capital outflow decreases.
12. In which of the following situations must national saving rise?
a. Both domestic investment and net capital outflow increase.
b. Domestic investment increases and net capital outflow
decreases.
c. Domestic investment decreases and net capital outflow
increases.
d. Net exports decrease and domestic investment is unchanged.
13. The country of Freedonia has a GDP of $2,000, consumption of
$1,200, and government purchases of $400. This implies that it has
a. national saving of $400.
Chapter 31: Open-Economy Macroeconomics: Basic Concepts
143
b. investment plus net capital outflow of $400.
c. investment plus net exports of $400.
d. All of the above.
14. The exchange rate is about 153 Kazakhstan tenge per dollar.
According to purchasing-power parity, this exchange rate would rise
if
a. the price level in either the United States or Kazakhstan rose.
b. the price level in either the United States or Kazakhstan fell.
c. the price level in the United States rose or the price level in
Kasakhstan fell.
d. the price level in the United States fell or the price level in
Kasakhstan rose.
15. Suppose that the real exchange rate between the United States and
Kenya is defined in terms of baskets of goods. Which of the following
will increase the real exchange rate (that is increase the number of
baskets of Kenyan goods a basket of U.S. goods buys)?
a.
an increase in the number of Kenyan shillings that can be
Chapter 31: Open-Economy Macroeconomics: Basic Concepts
144
purchased with a dollar
b.
an increase in the price of U.S. baskets of goods
c.
a decrease in the price in Kenyan shillings of Kenyan goods
d.
All of the above are correct.
16. Which of the following could be a consequence of an appreciation of
the U.S. real exchange rate?
a. John, a French citizen, decides that US pork has become too
expensive and cancels his order.
b. Nick, a U.S. citizen, decides that his trip to Nepal would be too
costly and cancels his trip.
c. Roberta, a U.S. citizen, decides to import fewer windshield
wipers for her auto parts company.
d. All of the above are correct.
17. If purchasing-power parity holds, then the value of the
a. real exchange rate is equal to one.
b. nominal exchange rate is equal to one.
c. real exchange rate is equal to the nominal exchange rate.
Chapter 31: Open-Economy Macroeconomics: Basic Concepts
145
d. real exchange rate is equal to the difference in inflation rates
between the two countries.
18. When a country's central bank increases the money supply, its
a. price level rises and its currency appreciates relative to other
currencies in the world.
b. price level rises and its currency depreciates relative to other
currencies in the world.
c. price level falls and its currency appreciates relative to other
currencies in the world.
d. price level falls and its currency depreciates relative to other
currencies in the world.
19. How much of US dollar can be exchanged from 50.000 VND. Given that
the Exchange rate is 23.000VND/ dollar.
A. $2.17
B. $0.46
C. None of the above are correct
D. $3
Chapter 31: Open-Economy Macroeconomics: Basic Concepts
146
20. Exchange rate is 23.000 VND / Dollar. Suppose that a bottle of Heinz
ketchup is $3 in the US. According to purchasing power parities, how many
bottles of Heinz ketchup could be bought in Vietnam with $3?
A. 1
B. 2
C. 3
D. Cannot be determined because the price of a bottle of Heinz ketchup in
Vietnam is missing
ANSWER AND EXPLANATION
1. ANSWER: d. None of the above is correct.
(Explain: Oceania imports $100 and exports $40 => Oceania’s Net Export = $60
Escudia imports $40 and exports $100 => Escudia’s Net Export = $60)
2. ANSWER: d. $52.4 billion of exports and $44.1 billion of imports, and a
trade surplus
Chapter 31: Open-Economy Macroeconomics: Basic Concepts
147
(Explain: Net Export = Export - Import
⇔ $8.3 billion = Export - $52.4 billion
⇔ Export = $52.4 billion
Since Net Export = $8.3 billion > 0, the government is running budget surplus)
3. Answer: D. Trade balance is Net Export, balanced trade is when Net Export
= 0.
(Explain: Definition of Trade Balance and Balanced Trade)
4. ANSWER: a. investment for Larry and U.S. foreign direct investment.
(Explain: When Larry purchases a bookstore, he is investing (See PE text book
26-2b p.556 for more details). When Larry opens a bookstore, he is actively
managing his investment, so his action is Foreign Direct Investment)
5. ANSWER: a. increases both U.S. net exports and U.S. net capital outflow.
(Explain: The US exports railroad engines to Bolivia, so US Net Export
increases. When the US receives the money from Bolivia, the US will use the
money to buy Bolivian assets because they were paid in Bolivian currency.
When the US buys foreign assets, US Net Capital Outflow increases.)
6. ANSWER: a. foreign assets by domestic residents minus the purchase of
domestic assets by foreign residents.
Chapter 31: Open-Economy Macroeconomics: Basic Concepts
148
(Explain: Theoretical Information)
7. ANSWER: A. (i), (iii), (iv)
(Explain: An action is considered as Foreign Direct Investment when the
investors actively manage their investment. A common example is opening
stores in other countries. So (i), (iii), and (iv) are Foreign Direct Investment.
An action is considered as Foreign Portfolio Investment when the
investors have a passive role in their investment. A common example is buying
stocks and bonds issued by foreign companies. So (ii) and (v) are Foreign
Portfolio Investment.)
8. ANSWER: c. $100 million and domestic investment is $70 million.
(Explain: Net Export = Net Capital Outflow = $100 million
Saving = Investment + Net Capital Outflow
⇔ Investment = $70 million)
9. ANSWER: a. both positive net exports and positive net capital outflow.
(Explain: When a country’s goods is attractive to other countries. Others will
buys goods from that country. As a result, NX is positive.
That country will use foreign currency to buy foreign assets. As a result,
Purchase of foreign assets by domestic residents increases => NCO is positive)
Chapter 31: Open-Economy Macroeconomics: Basic Concepts
149
10. ANSWER: c. 1.30 Canadian dollars per U.S. dollar.
(Explain: Real exchange rate =
𝑁𝑜𝑚𝑖𝑛𝑎𝑙 𝐸𝑥𝑐ℎ𝑎𝑛𝑔𝑒 𝑟𝑎𝑡𝑒 ×𝑈𝑆 𝑝𝑟𝑖𝑐𝑒
𝐶𝑎𝑛𝑎𝑑𝑖𝑎𝑛 𝑝𝑟𝑖𝑐𝑒
⇔ Nominal Exchange rate = 1.3 Canadian dollar / US dollar)
11. ANSWER: d. decrease, and U.S. net capital outflow decreases.
(Explain: The US is importing sardines from Morocco => US Net Export
decreases. Since the US paid Morocco with US dollar, Morocco can use that
amount of US dollar to buy US asset. So, Purchase of US assets by Moroccan
increases => US Net Capital Outflow decreases)
12. ANSWER: a. Both domestic investment and net capital outflow increase.
(Explain: Saving = Investment + Net Capital Outflow = Investment + Net
Export.
Without any exact numbers given, the situation where Saving must increase is
when both Investment and Net Capital Outflow (also Net Export) increase.)
13. ANSWER: d. All of the above.
(Explain: Y =$2000; C = $1200; G = $400
Y = C + I + G + NX
⇔ I + NX = I + NCO = Saving = $400)
Chapter 31: Open-Economy Macroeconomics: Basic Concepts
150
14. ANSWER: d. the price level in the United States fell or the price level in
Kasakhstan rose.
(Explain: Exchange rate =
𝐾𝑎𝑧𝑎𝑘ℎ𝑠𝑡𝑎𝑛 𝑡𝑒𝑛𝑔𝑒
𝐷𝑜𝑙𝑙𝑎𝑟
The exchange rate depends on the price level of both countries.
So, in order for Exchange rate to rise, Price level in the US must fall or price
level in Kazakhstan must increase.)
15. ANSWER: d. All of the above are correct.
(Explain: Real exchange rate (Kenya/US) =
𝑁𝑜𝑚𝑖𝑛𝑎𝑙 𝐸𝑥𝑐ℎ𝑎𝑛𝑔𝑒 𝑟𝑎𝑡𝑒 (𝑠ℎ𝑖𝑙𝑙𝑖𝑛𝑔𝑠/𝑑𝑜𝑙𝑙𝑎𝑟) ×𝑈𝑆 𝑝𝑟𝑖𝑐𝑒
𝐾𝑒𝑛𝑦𝑎 𝑝𝑟𝑖𝑐𝑒
a. When a dollar can buy more shillings, Nominal Exchange rate increases
=> Real exchange rate increases.
b. When US price of basket increases => Real Exchange Rate increases
c. When Kenyan price of basket decreases => Real Exchange Rate
increases)
16. ANSWER: a. John, a French citizen, decides that US pork has become too
expensive and cancels his order.
Chapter 31: Open-Economy Macroeconomics: Basic Concepts
151
(Explain: When appreciation in US real exchange rate occurs, 1 US dollar can
buy more foreign currency. As a result, foreigners who want to buy US goods
will be discouraged because of the higher price.)
17. ANSWER: a. real exchange rate is equal to one.
(Explain: Purchasing power parities states that an amount of currency has the
same amount of purchasing power everywhere.
Recall that Real exchange rate is the rate at which one good can be traded with
another. Since you can buy the same amount of good everywhere with the same
amount of currency, Real Exchange rate = 1)
18. ANSWER: b. price level rises and its currency depreciates relative to other
currencies in the world.
(Explain: When the central bank increases money supply, Inflation rate will rise
and the value of the currency will decrease. An amount of currency now can
buy less foreign currency than before, so we say that the currency depreciates
relative to other currencies in the world.)
19. Answer: A. $2.17
Chapter 31: Open-Economy Macroeconomics: Basic Concepts
(Explain: Exchange Rate = 23.000 VND/ Dollar =
𝑉𝑁𝐷
𝐷𝑜𝑙𝑙𝑎𝑟
152
=
50.000
𝐷𝑜𝑙𝑙𝑎𝑟
Dollar = $2.17)
20. Ans: A. 1
(Explain: According to the purchasing power parities, the same amount of
currency can buy the same amount of goods anywhere (Real Exchange Rate =
1). Since $3 can be used to buy 1 bottle of ketchup in the US, $3 can also be
used to buy 1 bottle of ketchup in Vietnam.)
Chapter 32: A Macroeconomic Theory of the Open Economy
153
Chapter 32:
A Macroeconomic Theory of the Open Economy
I/ Market for loanable fund
For an open economy:
S = I + NCO
Supply of loanable funds: National saving
Demand of loanable funds: National Investment and Net Capital Outflow.
S = I + NCO is that for every dollar a nation saves ( S increases), it can either
use it finance purchase of domestic capital ( I increases ) or foreign capital
(NCO increases).
NCO > 0: Net outflow of capital. Purchase of capital overseas add to the
Demand for Loanable Fund
NCO < 0: Net inflow of capital. Purchase of capital coming from abroad reduce
the Demand for Loanable Fund.
Savings, Investment, and NCO are all influenced by Real Interest rate. People
rely on Real Interest Rate to decide to amount to save and to decide whether to
choose domestic or foreign assets.
Chapter 32: A Macroeconomic Theory of the Open Economy
154
II/ Market for foreign currency exchange:
Participants in this market trade US dollar for foreign currency.
NX = NCO
Why NX = NCO?
Suppose when Net Export > 0, America is exporting more than importing. With
the foreign currency on hands, America will buy foreign assets ( NCO > 0) and
vice versa.
Supply: NCO supplied to buy foreign assets
Demand: Q of dollars used to buy US net export of goods and services
Chapter 32: A Macroeconomic Theory of the Open Economy
155
Net Export is affected by Real Exchange Rate ( When Real Exchange Rate
increases => The dollars appreciate => Goods in the US are more expensive =>
Less Export and more Import =>Net Export decreases and vice versa). But
NCO is unaffected by Real Exchange Rate, so Supply of dollars in the market of
foreign-currency exchange is vertical.
III/ Simultaneous Equilibrium in two Markets:
The market for loanable fund is created based on:
S = I + NCO
The market for foreign-currency exchange is created based on:
NX = NCO
=> NCO is the link between the two markets.
NCO depends on Real Interest Rate
Chapter 32: A Macroeconomic Theory of the Open Economy
From the link, we can figure out how 2 markets are connected to each other:
156
Chapter 32: A Macroeconomic Theory of the Open Economy
157
*How to draw:
Step 1: Determine the equilibrium in the market for loanable fund
Step 2: Determine the NCO through the equilibrium in the market for loanable
fund
Step 3: Determine the Supply of dollars in the market of foreign-currency
exchange through the amount of NCO
IV/ How Policies and Events Affect an Open Economy:
1. Government Budget Deficit:
Chapter 32: A Macroeconomic Theory of the Open Economy
158
- When the government’s spending > the government’s revenue →
government runs a budget deficit or negative public saving, which
decreases National saving (= Public Saving + Private Saving).
❖ In the Market for Loanable Fund:
National Saving is the Supply source. The decrease in National saving =>
Supply curve shift left → increases the real interest rate→ reduces NCO.
❖ In the Market for Foreign-Currency Exchange:
NCO is the Supply source. The decrease in NCO => Supply curve shift left →
Real Exchange Rate is appreciated → Export↓, Import↑ → NX↓.
2. Trade Policy:
Chapter 32: A Macroeconomic Theory of the Open Economy
159
Trade policy influences Q of goods and services that country exports or
imports.
+ Tariff: a tax on imported goods
+ Import Quota: a limit on the quantity of imported goods.
It is argued that Trade Policy would shrink the size of the trade deficit.
However,
When a tariff or an import quota is imposed:
+ In the Market for Foreign-Exchange Currency: ↓ Import → ↑ NX
(Export - Import) → Demand curve in Foreign-Currency Exchange
Market shift right → the Real Exchange Rate is appreciated.
+ In the Market for Loanable Fund: There is no affect, so the Real Interest
Rate is unchanged, which leads to no change in NCO => the Supply
curve in Foreign-Currency Exchange doesn’t shift.
➔ There is no change in NX.
Why at the end, NX doesn’t change?
Net Export here is the overall Trade Balance of the country. For example, when
the USA imposes an import quota on Japanese cars.
- Initially, Trade Policy decreases Import in car industries which lead to the
Real Exchange Rate to appreciate. => NX increases
- When the Dollars is appreciated, goods from the USA become more
expensive relative to goods from other country => Exports decrease,
Import increase => NX decreases
➔ Trade Policy do not change Overall Trade Balance.
Chapter 32: A Macroeconomic Theory of the Open Economy
160
3. Political Instability and Capital Flight:
Capital Flight: a large and sudden decrease of assets located in a
country.
For example: If people began to view Mexico as a less stable country than they
had previously thought, they decided to pull some of their assets out of Mexico
to move these funds to the other safer countries.
● In Mexico:
- Foreign purchase of domestic assets ↓ → NCO ↑ → NCO curve shift
right:
Chapter 32: A Macroeconomic Theory of the Open Economy
161
+ In the Market for Loanable Funds: Domestic people want to
borrow more to buy Foreign Asset => Demand for loanable Fund ↑
→ Real Interest Rate ↑.
+ In the Market for Foreign-Currency Exchange: An increase in
NCO → Supply curve shift right → Real Exchange Rate is
depreciated → NX ↑ (good or bad depends on the ability to
produce goods.)
● Effects are opposite for the surrounding countries
EXERCISE
Chapter 32: A Macroeconomic Theory of the Open Economy
162
1. In the open-economy macroeconomic model, the market for loanable
funds identity can be written as
a. S = I
b. S = NCO
c. S = I + NCO
d. S + I = NCO
2. Jack and Jill are co-owners of the U.S. firm Wells Petroleum. Jack
borrows money to build an oil well in Texas. Jill borrows money to build
an oil well in Venezuela.
a. Both Jack and Jill’s purchase of capital count as demand for loanable
funds in the U.S. market.
b. Neither Jack nor Jill’s purchase of capital count as demand for loanable
funds in the U.S. market.
c. Jack’s purchase of capital counts as demand for loanable funds in the
U.S. market; Jill’s purchase does not.
d. Jill’s purchase of capital counts as demand for loanable funds in the
U.S. market; Jack’s purchase does not.
Chapter 32: A Macroeconomic Theory of the Open Economy
163
3. The real exchange rate equals the relative
a. price of domestic and foreign currency.
b. price of domestic and foreign goods.
c. rate of domestic and foreign interest.
d. None of the above is correct.
4. Net capital outflow = net exports implies that
a. the supply of dollars equals the demand for dollars in the foreigncurrency exchange market.
b. national saving equals domestic investment.
c. the volume of exports equals the volume of imports.
d. All of the above are correct.
5. In the market for foreign-currency exchange in the open-economy
macroeconomic model, a higher U.S. real exchange rate makes
a. U.S. goods more expensive relative to foreign goods and reduces the
quantity of dollars supplied.
Chapter 32: A Macroeconomic Theory of the Open Economy
164
b. U.S. goods more expensive relative to foreign goods and reduces the
quantity of dollars demanded.
c. foreign goods more expensive relative to U.S. goods and reduces the
quantity of dollars supplied.
d. foreign goods more expensive relative to U.S. goods and reduces the
quantity of dollars demanded.
Use the figure below to answer the question 6 and 7:
Chapter 32: A Macroeconomic Theory of the Open Economy
165
6. Refer to the figure above. In the foreign-currency exchange market, the
effects of an increase in the budget surplus is illustrated as a move from g
to
a. g.
b. h.
c. i.
d. None of the above is correct.
7. Refer to the figure above. Starting from r1 and E3, an increase in the
budget deficit can be illustrated as a move to
a. r0 and E4.
b. r0 and E2.
c. r2 and E4.
d. r2 and E2.
8. If the United States imposes an import quota on clothing, U.S. exports
a. increase, U.S. imports increase, and U.S. net exports are unchanged.
b. increase, U.S. imports decrease, and U.S. net exports increase.
c. decrease, U.S. imports increase, and U.S. net exports decrease.
Chapter 32: A Macroeconomic Theory of the Open Economy
166
d. decrease, U.S. imports decrease, and U.S. net exports are unchanged.
9. Which of the following is the correct way to show the effects of a new
import quota?
a. shift the demand for loanable funds right, the supply of dollars for
foreign exchange right, and the demand for dollars left
b. shift the demand for loanable funds right, and the supply of dollars for
foreign exchange left
c. shift the demand for dollars for foreign exchange left
d. None of the above is correct.
10. If Kenya experienced capital flight, the supply of Kenyan schillings in
the foreign-currency exchange market would shift
a. left, which would make the real exchange rate of the Kenyan schilling
appreciate.
b. left, which would make the real exchange rate of the Kenyan schilling
depreciate.
c. right, which would make the real exchange rate of the Kenyan schilling
appreciate.
Chapter 32: A Macroeconomic Theory of the Open Economy
167
d. right, which would make the real exchange rate of the Kenyan schilling
depreciate.
11. In 2002 it looked like the Argentinean government might default on its
debt (which eventually it did), the open-economy macroeconomic model
predicts that this should have
a. raised Argentinean interest rates and caused the Argentinean currency
to appreciate.
b. raised Argentinean interest rates and caused the Argentinean currency
to depreciate.
c. lowered Argentinean interest rates and caused the Argentinean
currency to appreciate.
d. lowered Argentinean interest rates and caused the Argentinean
currency to depreciate.
12. The money supply in the market for foreign-currency exchange is
vertical because:
A. The amount of NCO represents the quantity of dollars supplied for
buying assets abroad and NCO is not affected by Real Exchange Rate
Chapter 32: A Macroeconomic Theory of the Open Economy
168
B. The amount of NCO represents the quantity of dollars demanded for
buying assets abroad and NCO is not affected by Real Exchange Rate
C. The amount of NCO represents the quantity of dollars demanded for
buying assets abroad and NCO is not affected by Real Interest Rate
D. The amount of NCO represents the quantity of dollars supplied for
buying assets abroad and NCO is not affected by Real Interest Rate
13. Supposed that Cambodia is experiencing Capital Flight. What will the
event affect Vietnam? Given that Vietnam is near Cambodia.
A. Vietnamese Real interest rate decreases, Real exchange rate increases.
B. Vietnamese Real interest rate increases, Real exchange rate decreases
C. Vietnamese Real interest rate decreases, Real exchange rate decreases
D. Vietnamese Real interest rate increases, Real exchange rate increases
ANSWER AND EXPLANATION
1. ANSWER: c. S = I + NCO
(Explain: Theoretical Information)
Chapter 32: A Macroeconomic Theory of the Open Economy
169
2. ANSWER: a. Both Jack and Jill’s purchase of capital count as demand for
loanable funds in the U.S. market.
EXPLAIN: Jack borrows money to build an oil well in Texas, so his purchase
is counted as Domestic Investment.
Jill borrows money to build an oil well in Venezuela, so her purchase is
counted as Net Capital Outflow.)
3. ANSWER: b. price of domestic and foreign goods.
(EXPLAIN: Real exchange is the rate at which one good is traded for another.
E.g.: The real exchange rate is 2 Trung Nguyên coffee / Highlands
Coffee. Which means that 2 units of Trung Nguyên coffee can be traded for 1
unit of Highlands Coffee.)
4. ANSWER: a. the supply of dollars equals the demand for dollars in the
foreign-currency exchange market.
(EXPLAIN: The equality of NX and NCO illustrates the equilibrium in the
market for foreign-currency exchange.
The Supply of dollars comes from NCO
The Demand for dollars comes from NX
Chapter 32: A Macroeconomic Theory of the Open Economy
170
)
5. ANSWER: b. U.S. goods more expensive relative to foreign goods and
reduces the quantity of dollars demanded.
(EXPLAIN: The real exchange rate in the market of foreign-currency exchange
is the quantity of US goods in relative with other nations’ goods (US
goods/other nations’ goods).
When Real Exchange Rate increases, US goods are now more expensive
relative to other goods, so now domestic buyers will buy more foreign goods,
foreign buyers will buy less US goods => Export decreases, Import increases =>
Net Export decreases => Quantity demanded for dollars decreases)
6. ANSWER: c. i.
Chapter 32: A Macroeconomic Theory of the Open Economy
171
(EXPLAIN: An increase in budget surplus shifts the supply curve in the market
of loanable fund to the right. Since this does not shift the curve in market for
foreign-currency exchange, the market move along the same curve => g to i)
7. ANSWER: c. r2 and E4.
(EXPLAIN: An increase in budget deficit causes the supply curve in market of
loanable fund shift to the left.
A leftward shift in supply curve in market for loanable fund increases the
Real interest rate from point c to b
A leftward shift in supply curve in market for loanable fund increases the
real exchange rate from point g to f, but do not shift the curve.)
8. ANSWER: d. decrease, U.S. imports decrease, and U.S. net exports are
unchanged.
(EXPLAIN: Import decreases (1) => Increases Net Export => Shifts Demand
curve in market for foreign-currency exchange to the right.
This causes an appreciation in real exchange rate, making US goods more
expensive relative to other nations’ goods => Export decreases, Import
increases (2)
From (1), (2) => We conclude that Import and Export decreases, but Net
Export stays the same
Chapter 32: A Macroeconomic Theory of the Open Economy
172
(See more in p.696 PE textbook))
9. ANSWER: d. None of the above is correct.
(EXPLAIN: An import quota is the same with tariff, they shifts the Demand
curve for market for foreign-currency exchange)
10. ANSWER: d. right, which would make the real exchange rate of the
Kenyan schilling depreciate.
(EXPLAIN: Capital Flight means a large and sudden reduction in demand for
assets in a country.
Kenya experiences capital flight, making Foreign Purchase of Domestic
assets decreases => Net Capital Outflow increases => Shifts the Demand curve
in market for loanable fund to the right
Capital flight also shifts Net Capital Outflow curve to the right, causing
the supply of dollars in market for foreign-currency exchange increases.
Chapter 32: A Macroeconomic Theory of the Open Economy
173
11. ANSWER: b. raised Argentinean interest rates and caused the Argentinean
currency to depreciate.
(EXPLAIN: When government declared default on its debt, the government is
said to be running a budget deficit.
The effect on 3 graphs is the same with budget deficit)
12. Ans: A. The amount of NCO represents the quantity of dollars supplied for
buying assets abroad and NCO is not affected by Real Exchange Rate
(Explain: Theoretical Information)
Chapter 32: A Macroeconomic Theory of the Open Economy
174
13. Ans: A. Vietnamese Real interest rate decreases, Real exchange rate
increases
(Explain: When a country is experiencing Capital Flight, surrounding countries
will have to bear effects which are opposite to what the Capital Flight country
are bearing.)
Chapter 33: Aggregate Demand and Aggregate Supply
175
Chapter 33:
Aggregate Demand and Aggregate Supply
Recession: a period of declining real incomes and rising unemployment.
Depression: a severe recession.
I- Three Key Facts about Economic Fluctuations:
1. Fact 1: Economic Fluctuations are Irregular and Unpredictable
- Fluctuations in the economy are often called the Business Cycle.
➔ Economic Fluctuations correspond to changes in Business Conditions.
+ Economic Expansion: real GDP grows rapidly, business is good.
+ Economic Contraction: real GDP falls during recession, business
has trouble.
- Even though Economic Fluctuations are called Business Cycle, they do
NOT follow a regular, predictable pattern. In fact, they are almost
impossible to predict.
2. Fact 2: Most Macroeconomic Quantities Fluctuate Together
Chapter 33: Aggregate Demand and Aggregate Supply
176
- When real GDP falls in recession, so do personal income, corporate
profits, consumer spending, investment spending, industrial production,
and so on.
- Although many macroeconomic variables fluctuate together, they
fluctuate by different amounts.
3. Fact 3: As Output Falls, Unemployment Rises
When firms choose to produce a smaller quantity of goods and services, they
lay off workers, expanding the pool of unemployed.
II- Explaining Short-Run Economic Fluctuations:
-
Classical theory (Classical Dichotomy and Monetary Neutrality)
describes the world in the long run but not in the short run.
- To explain the economy in the short-run, we use Aggregate Demand and
Aggregate Supply Model. Focus on the behaviour of two variables:
+ Output is measured by real GDP.
+ Price level is measured by CPI or GDP deflator.
Chapter 33: Aggregate Demand and Aggregate Supply
177
III- Aggregate-Demand Curve:
- Aggregate-Demand Curve: a curve that shows the quantity of goods and
services that households, firms, the government and customers abroad
want to buy at each price level.
Chapter 33: Aggregate Demand and Aggregate Supply
1. Why the Aggregate-Demand Curve Slopes Downward?
Y = C + I + G + NX
* Assume G fixed
* The Price Level and Consumption: The Wealth Effect
Price level falls → Real value rises → consumers feel Wealthier
→ Demand rises
* The Price Level and Investment: The Interest-Rate Effect
Price level falls → Real value rises → Household do not need to hold much
money → Lend Out (e.g. Buying interest-bearing bonds or depositing in an
178
Chapter 33: Aggregate Demand and Aggregate Supply
179
interest-bearing saving account) → Interest rate falls → Encourage greater
spending on investment goods → Demand for goods and services rises
* The Price Level and Net Exports: The Exchange-Rate Effect
Price level falls → Interest rate falls → Investors seek higher return by
investing abroad → Supply of dollars in Foreign-Currency Market rises →
Exchange rate falls → Dollars depreciate → Net Export rises → Demand for
goods and services rises.
2. Why the Aggregate-Demand Curve Might Shift?
- Shift Arising from Changes in Consumption (C):
+ Tax cut, stock market boom: People spend more → Consumption
rises → Aggregate-Demand Curve shifts right
+ Tax hike, Stock market decline: People spend less →
Consumption falls → Aggregate-Demand Curve shifts left
- Shift Arising from Changes in Investment (I):
+ Optimism, Interest rate falls, Money supply rises: Firm decided to
invest more → Aggregate-Demand Curve shifts right.
+ Pessimism, Interest rate rises, Money supply falls: Firm cut back
on Investment → Aggregate-Demand Curve shifts left.
Chapter 33: Aggregate Demand and Aggregate Supply
180
- Shift Arising from Changes in Government Purchases (G):
+ Government spends more: Demand rises → Aggregate-Demand
Curve shifts right
+ Government cut back on spending: Demand falls → AggregateDemand Curve shifts left
- Shift Arising from Changes in Net Export (NX):
+ Boom overseas, Exchange rate depreciation: Foreigners buy more
domestic goods → Aggregate-Demand Curve shifts right.
+ Recession overseas, Exchange rate appreciation: Foreigners buy
less domestic goods → Aggregate-Demand Curve shifts left.
IV/ Aggregate-Supply Curve:
Aggregate-Supply Curve: a curve that shows the quantity of goods and
services that firms choose to produce and sell at each price level.
1/ Long-run Aggregate-Supply Curve:
- Is vertical because in the long-run, monetary neutrality is applied.
Nominal variables like Price Level cannot affect real variables like
Unemployment rate.
Chapter 33: Aggregate Demand and Aggregate Supply
-
181
Depends on the supplies of labor, capital, natural resources, and the
available technology
- Natural Level of Output = Potential Output = Full-employment Output:
the production of goods and services of an economy when its
unemployment rate is at the normal rate.
2/ Why long-run Aggregate Supply curve shifts:
- Long-run Aggregate Supply curve shifts because of changes in
Determinant of Productivity.
- Labor rises → Aggregate-Supply Curve shifts right and vice versa
- Physical or Human Capital rises → Aggregate-Supply Curve shifts right
and vice versa
Chapter 33: Aggregate Demand and Aggregate Supply
182
- Natural Resources (weather, mineral) are advantageous → AggregateSupply Curve shifts right and vice versa
- Technological Knowledge (or events act like technology) rises →
Aggregate-Supply shifts right and vice versa
3/ Using Aggregate Demand and Long-Run Aggregate Supply to Depict
Long-Run Growth and Inflation:
Eventually, technological process and money supply will rise over time:
- Technological Process → Long-Run Aggregate-Supply Curve shifts right
- Money Supply rises → Aggregate-Demand Curve shifts right
Chapter 33: Aggregate Demand and Aggregate Supply
183
4/ Short-Run Aggregate-Supply Curve:
Why does it slope upward?
- Sticky-Wage Theory:
Employers and employees create a contract and agree on nominal wage
according to expected inflation in the future.
Chapter 33: Aggregate Demand and Aggregate Supply
184
When Inflation rate is lower than expected Inflation rate, real wage is
now larger than expected, production is now less profitable, so firms will
reduce quantity of output supplied.
- Sticky-Price Theory:
Firms announce their price according to the expected inflation rate in the
future.
When Inflation rate is lower than Expected Inflation rate, some firms will
adjust their price immediately, some will not because they don’t want to
incur cost of changing menus. These firms will have higher price than
others which will decrease their sales and decrease their production.
Vice versa.
- Misperceptions Theory:
When price level falls, price of the sellers will fall too. When their prices
fall, they may mistakenly think that their relative price has fallen, so they
will decrease production.
Vice versa.
Chapter 33: Aggregate Demand and Aggregate Supply
185
*We can express the upward slope of the Short-run Aggregate Supply
curve mathematically: (a is a positive constant depicting how much
output responds to unexpected change in price level)
5/ Shifts in Short-run Aggregate Supply curve:
Same effects with all that shift Long-Run Aggregate-Supply Curve: Amount of
labor, Human and Physical Capital, Natural Resources, and Technological
Knowledge
Chapter 33: Aggregate Demand and Aggregate Supply
186
- Expected Price level rises → SRAS shifts left
- Expected Price level falls → SRAS shifts right
E.g.: When Expected price level rises, Nominal wage for workers is now higher
=> Costs of production is now higher => Production decreases at any given
price level => SRAS shifts left.
IV/ Economics Fluctuation:
1/ Effect of shift in Aggregate-Demand Curve:
The long-run equilibrium: At the equilibrium price, expected price level
equals actual price level.
Chapter 33: Aggregate Demand and Aggregate Supply
187
Suppose that the stock market crashes. In the short-run, this will shifts the
Aggregate Demand curve to the left, decreasing equilibrium price level and
natural level of output.
In the long-run, with the lower price level, people will start to adjust their
expected price level. They will decrease their expected price level, shifting the
Short-run Aggregate Supply to the right, Natural level of output is back to its
normal rate but Price level is now lower.
*Note: Policymakers can shifts Aggregate Demand curve to ease the effect of a
particular event.
Chapter 33: Aggregate Demand and Aggregate Supply
188
If left alone, the economy will adjust itself like above, Natural level of
output is unaffected in the long-run, but not the Price level. If the policymakers
intervent, they will try to shift Aggregate Demand curve so that both Natural
level of output and Price level is unaffected.
2/ Effect of shift in Aggregate Supply curve:
Suppose that the economy experience an event that increases costs of
production. In the short-run, such event will reduce production and shift Shortrun Aggregate Supply curve to the left, decreasing Natural level of Output and
increasing Price level (Stagflation: decreasing output and increasing price level)
In the long-run, people will adjust their expected price level. When
workers increase their expected price level and ask to raise their nominal wage,
Short-run Aggregate Supply curve will shift further to the left, firms’ costs will
rise even more, creating a never-ending loop.
But, eventually, the loop will slow down due to the limited bargaining
power of workers when unemployment rate is high. Nominal wage will rise,
firm’s costs will be reduced, and the economy shifts back to its initial
equilibrium.
Chapter 33: Aggregate Demand and Aggregate Supply
Policymakers can prevent this from happen by shifting Aggregate
Demand curve.
189
Chapter 33: Aggregate Demand and Aggregate Supply
190
EXERCISE
1.
Business cycles
a. are easily predicted by competent economists.
b. have never occurred very close together.
c. can only be seen as changes in real GDP.
d. None of the above is correct.
2.
The model of aggregate demand and aggregate supply explains the
relationship between
a. the price and quantity of a particular good.
b. unemployment and output.
c. wages and employment.
d. real GDP and the price level.
Chapter 33: Aggregate Demand and Aggregate Supply
3.
191
Which of the following is not included in aggregate demand?
a. purchases of stock and bonds
b. purchases of services such as visits to the doctor
c. purchases of capital goods such as equipment in a factory
d. purchases by foreigners of consumer goods produced in the United
States
4.
If the price level rises, the value of money
a. rises while foreign exchange rates and interest rates rise.
b. rises while foreign exchange rates and interest rates fall.
c. falls while foreign exchange rates and interest rate rise.
d. falls while foreign exchange rates and interest rates fall.
5.
If the price level falls, households
a. increase foreign bond purchases, and the supply of dollars in the
market for foreign-currency exchange increases.
Chapter 33: Aggregate Demand and Aggregate Supply
192
b. increase foreign bond purchases, and the supply of dollars in the
market for foreign-currency exchange decreases.
c. decrease foreign bond purchases, and the supply of dollars in
market for foreign-currency exchange increases.
d. decrease foreign bond purchases, and the supply of dollars in the
market for foreign-currency exchange decreases.
6.
An increase in the price level induces people to hold
a. less money, so they lend less, and the interest rate rises, so firms
increase investment.
b. less money, so they lend more, and the interest rate falls, so firms
decrease investment.
c. more money, so they lend more, and the interest rate falls, so firms
increase investment.
d. more money, so they lend less, and the interest rate rises, so firms
decrease investment.
7.
Suppose a stock market crash makes people feel poorer. This
decrease in wealth would induce people to
Chapter 33: Aggregate Demand and Aggregate Supply
a. decrease consumption, which shifts aggregate supply left.
b. decrease consumption, which shifts aggregate demand left.
c. increase consumption, which shifts aggregate supply right.
d. increase consumption, which shifts aggregate demand right.
8.
Which of the following shifts aggregate demand to the right?
a. the Fed sells bonds in the open market.
b. technological progress that increases the profitability of capital
goods.
c. the government cuts military expenditures.
d. a increase in the price level
9.
If people want to save more for retirement
a. or if the government raises taxes, aggregate demand shifts right.
b. or if the government raises taxes, aggregate demand shifts left.
193
Chapter 33: Aggregate Demand and Aggregate Supply
194
c. aggregate demand shifts right. If the government raises taxes,
aggregate demand shifts left.
d. aggregate demand shifts left. If the government raises taxes,
aggregate demand shifts right.
10.
If the dollar appreciates because of speculation or government policy
a. or other countries experience recessions, aggregate demand shifts
right in the United States.
b. or other countries experience recessions, aggregate demand shifts
left in the United States.
c. aggregate demand shifts right in the United States. If other
countries experience recessions aggregate demand shifts left in the
United States.
d. aggregate demand shifts left in the United States. If other countries
experience recessions aggregate demand shifts right in the United
States.
Chapter 33: Aggregate Demand and Aggregate Supply
11.
195
The long-run aggregate supply curve would shift right if immigration
from abroad
a. increased or Congress made a substantial increase in the minimum
wage.
b. decreased or Congress abolished the minimum wage
c. increased or Congress abolished the minimum wage.
d. decreased or Congress made a substantial increase in the minimum
wage.
12.
The long-run aggregate supply curve would shift right if the
government were to
a. increase the minimum-wage law.
b. make unemployment benefits more generous.
c. raise taxes on investment spending.
d. None of the above is correct.
13.
According to the aggregate demand and aggregate supply model, in
the long run an increase in the money supply leads to
Chapter 33: Aggregate Demand and Aggregate Supply
196
a. increases in both the price level and real GDP.
b. an increase in real GDP but does not change the price level.
c. an increase in the price level but does not change real GDP.
d. does not change either the price level or real GDP.
14.
The misperceptions theory of the short-run aggregate supply curve
says that output supplied will increase if the price level
a. increases less than expected so that firms believe the relative price
of their output has increased.
b. increases less than expected so that firms believe the relative price
of their output has decreased.
c. increases more than expected so that firms believe the relative price
of their output has increased.
d. increases more than expected so that firms believe the relative price
of their output has decreased.
Chapter 33: Aggregate Demand and Aggregate Supply
15.
197
According to the sticky wage theory of the short-run aggregate
supply curve if workers and firms expected prices to rise by 2 percent but
instead they rise by 3 percent, real wages
a. rise, so firms will hire more workers.
b. rise, so firms will hire fewer workers.
c. fall, so firms will hire more workers.
d. fall, so firms will hire fewer workers.
16.
The sticky price theory of the short-run aggregate supply curve says
that when prices fall unexpectedly, some firms will have
a. lower than desired prices which increases their sales.
b. lower than desired prices which depresses their sales.
c. higher than desired prices which increases their sales.
d. higher than desired prices which depresses their sales.
Chapter 33: Aggregate Demand and Aggregate Supply
17.
198
Assuming that a is positive, theories of short-run aggregate supply
are expressed mathematically as
a. quantity of output supplied = natural rate of output + a(actual price
level – expected price level).
b. quantity of output supplied = natural rate of output + a(expected
price level – actual price level).
c. quantity of output supplied = a(actual price level -expected price
level) – natural rate of output.
d. quantity of output supplied = a(expected price level – actual price
level) – natural rate of output.
18.
An increase in the expected price level shifts short-run aggregate
supply to the
a. right, and an increase in the actual price level shifts short-run
aggregate supply to the right.
b. right, and an increase in the actual price level does not shift shortrun aggregate supply.
Chapter 33: Aggregate Demand and Aggregate Supply
199
c. left, and an increase in the actual price level shifts short-run
aggregate supply to the left.
d. left, and an increase in the actual price level does not shift shortrun aggregate supply.
19.
Which of the following would cause prices to rise and real GDP to
fall in the short run?
a. an increase in the expected price level
b. an increase in the money supply
c. an increase in the quantity of labor available
d. All of the above are correct.
Chapter 33: Aggregate Demand and Aggregate Supply
Figure 1
20. Refer to Figure 1. If the economy starts at A and moves to D, the
economy moves
a. to A in the long run.
b. to B in the long run.
c. to C in the long run.
d. back to D in the long run.
200
Chapter 33: Aggregate Demand and Aggregate Supply
21.
201
Policymakers who control monetary and fiscal policy and want to
offset the effects on output of a leftward shift in aggregate supply could use
policy to shift
a. aggregate supply to the right.
b. aggregate supply to the left.
c. aggregate demand to the right.
d. aggregate demand to the left.
22.
When production costs rise, in the short run
a. output and prices rise.
b. output rises and prices fall.
c. output falls and prices rise.
d. output and prices fall.
Chapter 33: Aggregate Demand and Aggregate Supply
23.
202
Suppose the economy is in long-run equilibrium. If there is a sharp
decline in the stock market combined with a decrease in cost of production,
then we would expect that in the short run,
a. real GDP will rise and the price level might rise, fall, or stay the
same. In the long-run, real GDP will rise and the price level might
rise, fall, or stay the same.
b. the price level will fall, and real GDP might rise, fall, or stay the
same. In the long-run, real GDP and the price level will be
unaffected.
c. the price level will rise, and real GDP might rise, fall, or stay the
same. In the long run, real GDP will rise and the price level will
fall.
d. the price level will fall, and real GDP might rise, fall, or stay the
same. In the long run, real GDP will rise and the price level will
fall.
ANSWER AND EXPLANATION
1. ANSWER: d. None of the above are correct.
(Explanation: Business Cycles are Economic Fluctuation.
Chapter 33: Aggregate Demand and Aggregate Supply
203
Economic Fluctuations are unpredictable.
Economic Fluctuations are irregular, so it can occurred very close
together.
Economic Fluctuation can also be seen as changes in Unemployment
Rate and Investment Spending.
Read 33-1a PE textbook p.708 for more details)
2. ANSWER: d. real GDP and the price level.
(Explanation: The model of aggregate demand and aggregate supply depicts
the relationship between real GDP (quantity of output) and GDP deflator (Price
level))
3. ANSWER: a. purchases of stock and bonds.
(Explanation: Aggregate Demand includes the quantity of goods and services
households, firms, government, and customers abroad want to buy at each level.
So B, C, and D is included in the Aggregate Demand. Stock and bonds,
however, are assets, so their purchase are not included in Aggregate Demand)
4. ANSWER: c. falls while foreign exchange rates and interest rate rise.
(Explanation:
Chapter 33: Aggregate Demand and Aggregate Supply
204
According the the Wealth effect, Interest-rate effect, and Exchange rate
effect, increase in price level will lead to lower value of money, higher interest
rate and exchange rate)
5. ANSWER: a. increase foreign bond purchases, and the supply of dollars in
the market for foreign-currency exchange increases.
(Explanation: Exchange rate effect:
When the price level falls, interest rate decreases. Households will
increase foreign bonds purchases because they have a higher interest rate. To
buy foreign bonds, people will try to convert more dollars into foreign currency.
Thus, to supply of dollars in the market for foreign currency exchange
increases.)
6. ANSWER: d. more money, so they lend less, and the interest rate rises, so
firms decrease investment
(Explanation: An increase in the price level cause the value of money to
decreases => Consumers will feel poorer. They tend to hold more money and
lend out less. Interest rate will rises, making borrowing money from the bank
more expensive than before. So firms will decrease investment.)
Chapter 33: Aggregate Demand and Aggregate Supply
205
7. ANSWER: b. decrease consumption, which shifts aggregate demand left.
(Explanation: A stock market crash makes people feel poorer, so they will
spend less at any given price level => Decrease Consumption => Aggregate
Demand shifts left)
8. ANSWER: b. technological progress that increases the profitability of
capital goods.
(Explanation:
B is correct. When profitability of capital goods increases, firms will
purchase investment goods more => Investment increases => Aggregate
Demand shifts right)
9. ANSWER: b. or if the government raises taxes, aggregate demand shifts left.
(Explanation: When people want to save more for retirement, current
consumption must decrease => Consumption decreases => Aggregate Demand
shifts left.
When the government raises tax, consumption is more costly now, so
people will be less likely to consume => Consumption decreases => Aggregate
Demand shifts left)
Chapter 33: Aggregate Demand and Aggregate Supply
206
10. ANSWER: b. or other countries experience recessions, aggregate demand
shifts left in the United States.
(Explanation: When the dollar appreciates relative to other currencies, goods in
the US will be more expensive, so US people will buy more foreign goods and
foreigners will buy less US goods => Export decreases, Import increases => Net
Export decreases => Aggregate Demand shifts left
When other countries experience recessions, they will buy fewer goods in
the US => Export decreases => Net Export decreases => Aggregate Demand
shifts left)
11. ANSWER: c. increased or Congress abolished the minimum wage.
(Explanation: When immigration increases, the supply of labor increases =>
Long-run Aggregate Supply curve shifts right.
When the government lifts off minimum wage law, there would be less
unemployment => Supply of labor increases => Long-run Aggregate Supply
curve shifts right)
12. ANSWER: d. None of the above is correct.
(Explanation:
Chapter 33: Aggregate Demand and Aggregate Supply
207
A is wrong. When the government increases the minimum-wage law=>
Supply of labor decreases => Long-run Aggregate Supply curve shifts left
B is wrong. When the government makes unemployment benefits more
generous => Supply of labor decreases => Long-run Aggregate Supply curve
shifts left
C is wrong. When the government raises tax on spending => Decrease
Consumption => Aggregate Demand curve shifts left, not Long-run Aggregate
Supply curve.)
13. ANSWER: c. an increase in the price level but does not change real GDP.
(Explanation: An increase in money supply will increase the price level. But in
the long-run, price level does not affect the determinants of real GDP => So real
GDP does not change)
14. ANSWER: c. increases more than expected so that firms believe the
relative price of their output has increased.
(Explanation: When price level increases more than expected and the suppliers
see that their price of products rises. They believe that their relative price has
increased, so they will increase production)
15. ANSWER: c. fall, so firms will hire more workers.
Chapter 33: Aggregate Demand and Aggregate Supply
208
(Explanation: When price level rises more than expected, real wages decreases.
As a result, firms’ managers actually pay less for workers because the contracts
on wages was determined based on expected price level and cannot be changed
now. Production is now more profitable and they will hire more workers.)
16. ANSWER: d. higher than desired prices which depresses their sales.
(Explanation: When price falls unexpectedly, price of goods adjusts more
slowly or does not adjust because of menu cost. As a result, some firms will
have price higher than desired => Their sales will decrease)
17. ANSWER: a. quantity of output supplied = natural rate of output +
a(actual price level – expected price level).
(Explanation: Theoretical Information)
18. ANSWER: d. left, and an increase in the actual price level does not shift
short-run aggregate supply.
(Explanation: According to the formula,
When the expected price level increases, quantity of output supplied will
decrease => Short-run Aggregate Supply curve will shift left.
Chapter 33: Aggregate Demand and Aggregate Supply
209
The increase in actual price level does not shift Short-run Aggregate
Supply curve.)
19. ANSWER: a. an increase in the expected price level.
(Explanation: Consider this model
In order for the price level to rise and real GDP to fall in the short-run, the
equilibrium point A must move up and to the left. We can see that it can only be
achieved when the Short-run Aggregate Supply curve shift left.
A is correct. According to the formula below, an increase in the expected
price level cause the quantity of output supplied (Real GDP) to decrease =>
Short-run Aggregate Supply curve shift left.
Chapter 33: Aggregate Demand and Aggregate Supply
210
B is wrong. An increase in money supply will shift Aggregate Demand
curve to the right, increasing both Price level and quantity of output.
C is wrong. An increase in supply of labor cause the Short-run Aggregate
Supply curve shifts to the right.)
20. ANSWER: c. to C in the long run.
(Explanation: In the short-run, when the economy move from point A to D, it
reduces the price level. When the price level decreases, in the long run, people
will re-adjust their expectation of the price level. As the expected price level
decreases, Short-run Aggregate Supply curve shifts right)
21. ANSWER: c. aggregate demand to the right.
(Explanation: Policymakers will always want the economy to shift back to its
initial equilibrium point.
Chapter 33: Aggregate Demand and Aggregate Supply
211
Short-run Aggregate Supply curve shifts to the left. Policymakers will use
monetary and fiscal policies to shifting the Aggregate Demand curve to the right
to get back to the initial equilibrium)
22. ANSWER: c. output falls and prices rise.
(Explanation: When production costs rise, production is now less profitable
than before, so Short-run Aggregate Supply curve will shift to the left,
decreasing output and increasing price level.
Chapter 33: Aggregate Demand and Aggregate Supply
212
)
23. ANSWER: d. the price level will fall, and real GDP might rise, fall, or stay
the same. In the long run, real GDP will rise and the price level will fall.
(Explanation:
In the short-run:
+ Stock market declined causes the Aggregate Demand curve to shift to the
left from AD1 to AD2, moving the equilibrium point from A to B
Chapter 33: Aggregate Demand and Aggregate Supply
213
+ A decrease in cost of production will shift the Short-run Aggregate
Supply curve to the right from SRAS1 to SRAS2, moving the equilibrium
point from B to C.
=> In the short-run, Price level will decrease, but Real GDP will be ambiguous.
Real GDP can decrease, increase, or stay the same depending on the magnitude
of the shift in Short-run Aggregate Supply curve and Aggregate Demand curve.
From the short-run, we can see that Price level decreases. In the long-run,
people will start adjusting their expected price level. As expected price level
decreases, Short-run Aggregate Supply curve shifts right from SRAS2 to
SRAS3, moving the equilibrium point from C to D, resulting in a decrease in
Price level and an increase in Real GDP.
Chapter 33: Aggregate Demand and Aggregate Supply
214
CHAPTER 34: The Influence of Monetary and Fiscal Policy on Aggregate Demand
215
Chapter 34:
The Influence of Monetary and Fiscal Policy on Aggregate
Demand
I- How Monetary Policy influences Aggregate Demand
1. The theory of liquidity preference
● Theory of liquidity preference (Keynes’s theory):
- A simple theory of the interest rate (denoted r).
- The interest rate r adjusts to bring money supply and money
demand into balance.
- Theory of liquidity preference explains both the nominal interest
rate and real interest rate.
● Money supply: assume fixed by the central bank, does not depend on
interest rate.
- Alter the money supply by changing the quantity reserves
CHAPTER 34: The Influence of Monetary and Fiscal Policy on Aggregate Demand
216
- The Fed can influence the money supply using a variety of other
tools
● Money demand reflects how much wealth people want to hold in liquid
form.
- Recall that any asset’s liquidity refers to the ease with which that
asset can be converted into the economy’s medium of exchange.
- For simplicity, suppose household wealth includes only two assets:
+ Money - liquid but pay no interest
+ Bonds - pay interest but not as liquid
- A household’s “money demand” reflects its preference for liquidity
- The variables that influence money demand: Y, r, P.
+ An increase in the interest rate ⇒ raises the cost of holding
money ⇒ reduces money demanded.
+ A decrease in the interest rate ⇒ reduces the cost of holding
money ⇒ raises money demanded
+ Y (real income) increases ⇒ increases money demanded,
other things equal
+ P (price level) increases ⇒ increases money demanded,
other things equal
CHAPTER 34: The Influence of Monetary and Fiscal Policy on Aggregate Demand
2. The downward slope of the Aggregate Demand curve
217
CHAPTER 34: The Influence of Monetary and Fiscal Policy on Aggregate Demand
218
- The interest-rate effect can be summarized in three steps:
(1) A higher price level raises money demand.
(2) Higher money demand leads to a higher interest rate.
(3) A higher interest rate reduces the quantity of goods and services
demanded.
- The same logic works for a decline in the price level
3. Change in money supply
- To achieve macroeconomic goals, the FED can use monetary policy to
shift the AD curve
CHAPTER 34: The Influence of Monetary and Fiscal Policy on Aggregate Demand
219
- The FED’s policy instrument is Money Supply
To sum up:
- When the Fed increases/ contracts the money supply
⇒ It lowers/raises the interest rate
⇒ increases/reduces the quantity of goods and services demanded for any
given price level
⇒ shifts the aggregate-demand curve to the right/left
4. The role of interest-rate target and FED policy
CHAPTER 34: The Influence of Monetary and Fiscal Policy on Aggregate Demand
220
- The news often report that the FED targets the interest rate. More
precisely, the federal funds rate, which banks charge each other on shortterm loans
- To change the interest rate and shift the AD curve, the FED conduct open
market operation.
- Monetary policy can be described either in terms of the money supply or
in terms of the interest rate.
- In general: Changes in monetary policy aimed at
Expanding aggregate demand can be described either as increasing the money
supply or as lowering the interest rate and vice versa.
II- How fiscal policy influences aggregate demand
1. Fiscal policy: the setting of the level of govt spending and taxation by govt
policymakers.
- Expansionary fiscal policy
+ an increase in G (government spending) and/or decrease in T
(taxation)
CHAPTER 34: The Influence of Monetary and Fiscal Policy on Aggregate Demand
221
+ shifts AD right
- Contractionary fiscal policy
+ a decrease in G (government spending) and/or increase in T
(taxation)
+ shifts AD left
- Fiscal policy has two effects on AD:
+ The multiplier effect: the additional shifts in aggregate demand
that result when expansionary fiscal policy increases income and
thereby increases consumer spending.
For example: If the govt buys $20b of planes from Boeing, Boeing’s
revenue increases by $20b. This is distributed to Boeing’s workers (as wages)
and owners (as profits or stock dividends). These people are also consumers
and will spend a portion of the extra income. This extra consumption causes
further increases in aggregate demand.
CHAPTER 34: The Influence of Monetary and Fiscal Policy on Aggregate Demand
222
2. The multiplier effect
- Multiplier effect: the additional shifts in AD that result when fiscal
policy increases income and thereby increases consumer spending
⇒ The size of the multiplier effect depends on how much consumers
respond to increases in income.
- Marginal propensity to consume (MPC): the fraction of extra income
that households consume rather than save.
Formula: Multiplier = 1/(1 − MPC).
CHAPTER 34: The Influence of Monetary and Fiscal Policy on Aggregate Demand
223
⇒ The size of the multiplier depends on MPC.
=> A bigger MPC means changes in Y cause bigger changes in C, which in turn
cause more changes in Y.
3. The crowding-out effect
● Crowding-out effect: the offset in aggregate demand that results when
expansionary fiscal policy raises the interest rate and thereby reduces
investment spending.
- The fiscal expansion raises the demand for money => Income increases
- Interest rate increases => reduces the quantity of goods and services demanded
=> Crowding-out effect partially offsets the impact of government purchases on
aggregate demand.
CHAPTER 34: The Influence of Monetary and Fiscal Policy on Aggregate Demand
224
⇒ Sum up: When the government increases its purchases, the aggregate
demand for goods and services could rise by more or less depending on the
sizes of the multiplier and crowding-out effects.
* The multiplier effect makes the shift in aggregate demand greater.
* The crowding-out effect pushes the aggregate-demand curve in the opposite
direction and, if large enough, could result in an aggregate-demand shift of less.
● Changes in tax
- A tax cut increases households’ take-home pay.
- Households respond by spending a portion of this extra income => AD
shifts right.
- The size of the shift is affected by the multiplier and crowding-out
effects.
- Whether households perceive the tax cut to be temporary or permanent.
CHAPTER 34: The Influence of Monetary and Fiscal Policy on Aggregate Demand
225
-> A permanent tax cut causes a bigger increase in C—and a bigger shift in
the AD curve—than a temporary tax cut.
III. Using policy to stabilize the economy:
1. The case for active stabilization policy
- Keynes:
“Animal spirits” cause waves of pessimism and optimism
among households and firms => shifts in AD and fluctuations in output
and employment.
● Other factors:
- Booms and recessions abroad
- Stock market booms and crashes
=> Policymakers do nothing, these fluctuations are destabilizing to businesses,
workers, consumers.
- Proponents of active stabilization policy believe the govt should use
policy to reduce these fluctuations:
CHAPTER 34: The Influence of Monetary and Fiscal Policy on Aggregate Demand
226
● When GDP falls below its natural rate => use expansionary
monetary or fiscal policy to prevent or reduce a recession.
● When GDP rises above its natural rate, use contractionary policy
to prevent or reduce an inflationary boom.
2. The case against active stabilization policy
❖ Monetary policy affects economy with a long lag:
- Firms make investment plans in advance, so I
takes time to
respond to changes in r.
- Most economists believe it takes at least 6 months for mon policy
to affect output and employment.
❖ Fiscal policy also works with a long lag:
- Changes in G and T require acts of Congress.
- The legislative process can take months or years.
❖ Due to these long lags, critics of active policy argue that such policies
may destabilize the economy rather than help it:
CHAPTER 34: The Influence of Monetary and Fiscal Policy on Aggregate Demand
227
- By the time the policies affect agg demand, the economy’s
condition may have changed.
❖ These critics contend that policymakers should focus on long-run goals
like economic growth and low inflation.
3. Automatic stabilizers
- Automatic stabilizers: changes in fiscal policy that stimulate aggregate
demand when the economy goes into a recession without policymakers
having to take any deliberate action.
- Examples for automatic stabilizers
● The tax system: In recession, taxes fall automatically, which
stimulates agg demand.
● Govt spending:
- In recession, more people apply for public assistance (welfare,
unemployment insurance).
CHAPTER 34: The Influence of Monetary and Fiscal Policy on Aggregate Demand
228
- Govt spending on these programs automatically rises, which
stimulates agg demand.
EXERCISE
1. Which of the following is not a response that would result from a
decrease in the price level and so help to explain the slope of the
aggregate demand curve?
a. When interest rates fall, Sleepwell Hotels decides to build some
new hotels.
b. The exchange rate falls, so French restaurants in Paris buy more
Iowa (US) pork.
c. Janet feels wealthier because of the price drop and so she decides
to remodel her bathroom.
CHAPTER 34: The Influence of Monetary and Fiscal Policy on Aggregate Demand
229
d. With prices down and wages fixed by contract, Gatekeeper
Computers decides to lay off workers.
2. Liquidity preference theory is most relevant to the
a. short run and supposes that the price level adjusts to bring money
supply and money demand into balance.
b. short run and supposes that the interest rate adjusts to bring money
supply and money demand into balance.
c. long run and supposes that the price level adjusts to bring money
supply and money demand into balance.
d. long run and supposes that the interest rate adjusts to bring money
supply and money demand into balance.
3. According to liquidity preference theory, the opportunity cost of
holding money is
a. the interest rate on bonds.
b. the inflation rate.
c. the cost of converting bonds to a medium of exchange.
d. the difference between the inflation rate and the interest rate on
bonds.
4. The opportunity cost of holding money
CHAPTER 34: The Influence of Monetary and Fiscal Policy on Aggregate Demand
230
a. decreases when the interest rate increases, so people desire to hold
more of it.
b. decreases when the interest rate increases, so people desire to hold
less of it.
c. increases when the interest rate increases, so people desire to hold
more of it.
d. increases when the interest rate increases, so people desire to hold
less of it.
Figure 1
5. Refer to Figure 1. Which of the following is correct?
CHAPTER 34: The Influence of Monetary and Fiscal Policy on Aggregate Demand
231
a. If the interest rate is 4 percent, there is excess money demand, and
the interest rate will fall.
b. If the interest rate is 3 percent, there is excess money supply, and
the interest rate will rise.
c. If the interest rate is 4 percent, the demand for goods will rise when
the money market is in its new equilibrium.
d. If the interest rate is 4 percent, the demand for goods will rise and
money supply will drop when the money market is in its new
equilibrium.
6. According to liquidity preference theory if the price level increases,
the equilibrium interest rate
a. rises so that the aggregate quantity of goods demand rises.
b. rises so that the aggregate quantity of goods demanded falls.
c. falls so that the aggregate quantity of goods demanded rises.
d. falls so that the aggregate quantity of goods demanded falls.
7. If the Fed conducts open-market sales, the money supply
a. increases and aggregate demand shifts right.
b. increases and aggregate demand shifts left.
CHAPTER 34: The Influence of Monetary and Fiscal Policy on Aggregate Demand
232
c. decreases and aggregate demand shifts right.
d. decreases and aggregate demand shifts left.
8. If the stock market crashes,
a. aggregate demand increases, which the Fed could offset by
increasing the money supply.
b. aggregate demand increases, which the Fed could offset by
decreasing the money supply.
c. aggregate demand decreases, which the Fed could offset by
increasing the money supply.
d. aggregate demand decreases, which the Fed could offset by
decreasing the money supply.
9. Fiscal policy refers to the idea that aggregate demand is changed by
changes in
a. the money supply.
b. government spending and taxes.
c. trade policy.
d. All of the above are correct.
CHAPTER 34: The Influence of Monetary and Fiscal Policy on Aggregate Demand
233
10. The government buys a bridge. The owner of the company that
builds the bridge pays her workers. The workers increase their
spending. Firms that the workers buy goods from increase their
output. This type of effect on spending illustrates
a. the multiplier effect.
b. the crowding-out effect.
c. the marginal propensity to consume effect.
d. None of the above is correct.
11. If the MPC = 3/5, then the government purchases multiplier is
a. 5/3.
b. 5/2.
c. 5.
d. 15.
12. Which of the following correctly explains the crowding-out effect?
a. An increase in government expenditures decreases the interest rate
and so increases investment spending.
b. An increase in government expenditures increases the interest rate
and so reduces investment spending.
CHAPTER 34: The Influence of Monetary and Fiscal Policy on Aggregate Demand
234
c. A decrease in government expenditures increases the interest rate
and so increases investment spending.
d. A decrease in government expenditures decreases the interest rate
and so reduces investment spending.
13. The economy is in long-run equilibrium. Advances in technology shift
the long-run aggregate supply curve $50 billion to the right.
Optimistic investors have shifted the aggregate demand curve $100
billion to the right. In order to stabilize the price level at its original
value, the government wants to reduce its spending. If the crowdingout effect is always half of the multiplier effect, and if the MPC equals
0.75, then the government must cut its spending by
a. $4 billion.
b. $25 billion.
c. $50 billion.
d. $100 billion.
14. If households view a tax cut as being temporary, the tax cut
a. has no affect on aggregate demand.
b. has more of an affect on aggregate demand than if households view
it as permanent.
CHAPTER 34: The Influence of Monetary and Fiscal Policy on Aggregate Demand
235
c. has the same affect as when households view the cut as permanent.
d. has less of an affect on aggregate demand than if households view
it as permanent.
Figure 2
15. Refer to Figure 2. If the economy is at point b, a policy to restore full
employment would be
a. an increase in the money supply.
b. a decrease in government purchases
c. an increase in taxes
d. All of the above are correct.
CHAPTER 34: The Influence of Monetary and Fiscal Policy on Aggregate Demand
236
ANSWER AND EXPLANATION
1. ANSWER: d. With prices down and wages fixed by contract, Gatekeeper
Computers decides to lay off workers.
(Explain:
In order to explain the downward slope of Aggregate Demand curve, we
rely on 3 effects: Wealth effect, Interest rate effect, and Exchange rate effect.
A is wrong. When interest rate falls, borrowing is now cheaper than before,
which encourages people to borrow more to invest more in capital goods =>
Interest-rate effect.
B is wrong. When exchange rate falls, one French currency can buy more US
currency => Goods in the US is cheaper => French restaurants buy more US
goods => Exchange rate effect.
C is wrong. When the price level drops, the value of each dollars increases =>
Janet will feel wealthier => the wealth effect.
D is correct. When prices decreases => Value of money increases. Since
Nominal Wages are fixed, workers now have earn more real value of money.
Thus, firms must lay off some workers. However, this does not represent 1 of 3
effects which explains the downward slope of Aggregate Demand curve.)
CHAPTER 34: The Influence of Monetary and Fiscal Policy on Aggregate Demand
237
2. ANSWER: b. short run and supposes that the interest rate adjusts to bring
money supply and money demand into balance.
(Explain: Theoretical Information)
3. ANSWER: a. the interest rate on bonds.
(Explain: Instead of holding money, you can use that amount of money to buy
interest-bearing bond and earn interest every period. So we say that the
opportunity cost of holding money is the interest rate on bonds.)
4. ANSWER: d. increases when the interest rate increases, so people desire to
hold less of it.
(Explain: When the interest rate increases, you will actually get more interest if
you buy bonds instead of holding money. So, as interest rate increases, the
opportunity cost of holding money increases, people will choose to hold less
money)
5. ANSWER: c. If the interest rate is 4 percent, the demand for goods will rise
when the money market is in its new equilibrium.
(Explain: Treat money market like any other market we have learned
previously.
CHAPTER 34: The Influence of Monetary and Fiscal Policy on Aggregate Demand
238
The correct answer is C. When interest rate is 4%, shortage occurs. So, the
market will eventually get back to its initial equilibrium. As Demand for Money
rise, Demand for goods also rises. Supply of money does not change even when
Shortage (or Surplus) presents because Money Supply is fixed by the the Fed)
6. ANSWER: b. rises so that the aggregate quantity of goods demanded falls.
(Explain: When Price level increases, people will demand for more money in
the Money Market => Shift Money Demand curve in the Money Market right,
which drives up interest rate. An increased interest rate discourages the
borrowing for money to invest => Aggregate Demand falls
7. ANSWER: d. decreases and aggregate demand shifts left.
CHAPTER 34: The Influence of Monetary and Fiscal Policy on Aggregate Demand
239
(Explain: If the Fed conducts open-market sales => Money Supply shifts lefts.
It raises interest rate and decreases demand for goods and services at any given
price level => Aggregate Demand shifts left)
8. ANSWER: c. aggregate demand decreases, which the Fed could offset by
increasing the money supply.
(Explain: When the stock market crashes => Aggregate Demand shifts left. The
Fed can offset the effect by controlling Money Supply to shift back Aggregate
Demand to the right)
9. ANSWER: b. government spending and taxes.
(Explain: Theoretical Information)
10. ANSWER: a. the multiplier effect.
(Explain: Theoretical Information)
11. ANSWER: b. 5/2.
(Explain: Multiplier =
1
1 − 𝑀𝑃𝐶
)
12. ANSWER: b. An increase in government expenditures increases the interest
rate and so reduces investment spending.
CHAPTER 34: The Influence of Monetary and Fiscal Policy on Aggregate Demand
240
(Explain: An increase in government expenditures shifts the Aggregate
Demand to the right, which indicates an increase in spending in goods and
services => Money Demand in the Money Market increases => Interest rate
increases => Investment spending decreases => Aggregate Demand shifts left.)
13. ANSWER: b. $25 billion.
(Explain: Long-run Aggregate Demand curve shifted by $50 billions,
Aggregate Demand curve shifted by $100 billions => In order to come back to
the equilibrium, Aggregate Demand curve must shift back $50 billions.
Denote A as the cut needed in government spending.
+ Multiplier Effect:
Total change in Demand = 4A
+ Crowding-out effect:
Crowding-out effect = 2A
$50 billions = Total change in Demand - Crowding-out Effect
⇔ A = $25 billions
CHAPTER 34: The Influence of Monetary and Fiscal Policy on Aggregate Demand
241
=> Government must cut its spending by $25 billions in order to get the price
back to the original value)
14. ANSWER: d. has less of an affect on aggregate demand than if households
view it as permanent.
(Explain: When the tax cut is temporary, households will view the extra income
from the tax cut as a small addition to their financial resources. Contrary to
temporary tax cut, permanent tax cut will induce households into viewing this
extra income as a large addition to their financial resources. So, temporary tax
cut has smaller effect on Aggregate Demand than permanent tax cut)
15. ANSWER: a. an increase in the money supply.
(Explain: Currently the economy is having AD2, which is lower than the longrun equilibrium Aggregate Demand => Decrease Production => Decrease
Employment. To regain full employment, AD2 must shift back to the long-run
equilibrium, which is back to AD1.
A is correct. An increase in Money Supply will shift AD right
B is wrong. A decrease in Government purchase will shift AD left
C is wrong. An increase in taxes will depress demand for goods and services =>
Shift AD left)
Chapter 35: The Short-Run Trade-off between Inflation and Unemployment
242
Chapter 35:
The Short-Run Trade-off between Inflation and Unemployment
I. The Phillips Curve:
1. Origins of the Phillips Curve:
- Phillips curve: a curve that shows the short-run trade-off between inflation and
unemployment
- It illustrates a negative association between the inflation rate and the unemployment
rate.
●
Low unemployment => high inflation
●
High unemployment => low inflation
- Since fiscal and monetary policy affect agg demand, the PC appeared to offer
policymakers a menu of choices:
●
Low unemployment with high inflation
●
Low inflation with high unemployment
●
Anything in between.
2. Aggregate Demand, Aggregate Supply, and the Phillips Curve:
Chapter 35: The Short-Run Trade-off between Inflation and Unemployment
243
- The Phillips curve shows the combinations of inflation and unemployment that arise
in the short run as shifts in the aggregate-demand curve move the economy along the
short-run aggregate-supply curve
- Natural-rate hypothesis: the claim that unemployment eventually returns to its
normal or “natural” rate in the long-run, regardless of the inflation rate
- The greater the expansion of the money supply, the faster AD will shift to the right,
resulting in a larger increase in prices
But this higher inflation will not produce lower unemployment in the long run,
unemployment always goes to its natural rate whether inflation is high or low.
In the long run, faster money growth only causes faster inflation.
II. Shifts in the Phillips Curve: the role of expectations
-
Long-Run Phillips Curve: vertical, price level and inflation do not affect
unemployment rate in the long run (follow classical theory)
●
Unemployment rate does not depend on money growth or inflation
❖ Reconciling Theory and Evidence:
-
Theory (Friedman and Phelps): PC is vertical in the long run.
Chapter 35: The Short-Run Trade-off between Inflation and Unemployment
-
244
A variable of the analysis of the inflation–unemployment trade-off: expected
inflation: expected inflation - a measure of how much people expect the price
level to change.
+ Expected price level affects nominal wages
+ Determines the position of the short-run aggregate-supply curve.
- The Phillips Curve Equation:
𝑈𝑛𝑒𝑚𝑝𝑙𝑜𝑦𝑚𝑒𝑛𝑡 𝑟𝑎𝑡𝑒 =
𝑛𝑎𝑡𝑢𝑟𝑎𝑙 𝑟𝑎𝑡𝑒 𝑜𝑓 𝑢𝑛𝑒𝑚𝑝𝑙𝑜𝑦𝑚𝑒𝑛𝑡 − 𝑎 × (𝑎𝑐𝑡𝑢𝑎𝑙 𝑖𝑛𝑓𝑙𝑎𝑡𝑖𝑜𝑛 − 𝑒𝑥𝑝𝑒𝑐𝑡𝑒𝑑 𝑖𝑛𝑓𝑙𝑎𝑡𝑖𝑜𝑛)
- Short run: Fed can reduce u-rate below the natural u-rate by making inflation
greater than expected.
+
There can be no stable short-run Phillips curve.
+
Each short-run Phillips curve reflects a particular expected rate of inflation.
+
Expected inflation increases → the short-run Phillips curve shifts
rightward and vice versa.
- Long run: Expectations catch up to reality, u-rate goes back to natural u-rate
whether inflation is high or low.
Chapter 35: The Short-Run Trade-off between Inflation and Unemployment
245
- Natural rate of unemployment is not necessarily socially desirable, not constant.
- The vertical long-run aggregate-supply curve and the vertical long-run Phillips
curve both imply that monetary policy influences nominal variables (the price level
and the inflation rate) not real variables (output and unemployment).
How Expected Inflation Shifts the Short-Run Phillips Curve
Example:
Chapter 35: The Short-Run Trade-off between Inflation and Unemployment
- Initially,
+ expected & actual inflation = 3%,
+ Natural unemployment rate (6%).
- Fed makes inflation 2% higher than expected, u-rate falls to 4%.
- In the long run,
+ expected inflation increases to 5%,
+ PC shifts upward,
+ Unemployment returns to its natural rate.
- Monetary policy does not affect natural rate of unemployment.
246
Chapter 35: The Short-Run Trade-off between Inflation and Unemployment
247
- Expansionary policy moves the economy up along the short-run Phillips curve,
expected inflation rises → short-run Phillips curve shifts right and vice versa.
❖ The Natural Experiment for the Natural-Rate Hypothesis
- Natural-rate hypothesis: unemployment eventually returns to its normal, or
natural, rate, regardless of the rate of inflation
III. Shifts in the Phillips Curve: The Role of Supply Shocks:
- Supply shock: an event that directly alters firms’ costs and prices, shifting the
economy’s aggregate supply curve and thus the Phillips curve
+ Short-run aggregate supply shifts left.
+ Short-run Phillips curve shifts right.
Example: large increase in oil prices → raises the cost of producing gasoline →
reduces the quantity of goods and services supplied.
As panel (a) shows, this reduction in supply is represented by the leftward shift in
the aggregate-supply curve from AS1 to AS2. Output falls from Y1 to Y2, and
the price level rises from P1 to P2. A combination of lower output and higher
price level results in a rightward shift in Short-run Phillips curve.
- Stagflation: The combination of falling output (stagnation) and rising prices
(inflation)
Chapter 35: The Short-Run Trade-off between Inflation and Unemployment
248
An Adverse Shock to Aggregate Supply
IV. The Cost of Reducing Inflation:
Disinflation: a reduction in the rate of inflation. Disinflation is different from Deflation
(a reduction in the price level). Disinflation is like slowing down, whereas Deflation is
like going in reverse.
❖ The Sacrifice Ratio:
- To reduce inflation, Fed must slow the rate of money growth, which reduces
aggregate demand.
+ Short run: Output falls and unemployment rises.
+ Long run: Output & unemployment return to their natural rates.
Chapter 35: The Short-Run Trade-off between Inflation and Unemployment
249
- Contractionary monetary policy moves economy from A to B.
- Over time, expected inflation falls, Phillips Curve shifts downward.
- In the long run, at point C: unemployment is back at its natural rate, but with
lower inflation.
Disinflation requires enduring a period of high unemployment and low output.
- Sacrifice Ratio: the number of percentage points of annual output lost in the
process of reducing inflation by 1 percentage point.
Chapter 35: The Short-Run Trade-off between Inflation and Unemployment
250
For example: Typical estimate of the sacrifice ratio: 5. Which means: to reduce
inflation rate 1%, must sacrifice 5% of a year’s output.
- Sacrifice can be spread out over time.
For example: To reduce inflation by 6%, can either
+ sacrifice 30% of GDP for one year
+ sacrifice 10% of GDP for three years
Rational Expectations and the Possibility of Costless Disinflation:
- Rational Expectation: the theory that people optimally use all the information
they have, including information about government policies, when forecasting
the future.
- Government
is
Credible:
people
adjust
expected
inflation
quickly,
unemployment rate gets back to its natural rate quicklier and vice versa.
- Low sacrifice ratio: shorter labor contracts, Central Bank is credible and vice
versa.
For example: Suppose the Fed successfully convinces everyone it is committed to
reducing inflation. Then expected inflation falls, the short-run Phillips Curve shifts
downward quicklier.
Chapter 35: The Short-Run Trade-off between Inflation and Unemployment
251
Result: Disinflations can cause less unemployment than the traditional sacrifice ratio
predicts when people find the government credible.
EXERCISE
1.
In the long run,
a. the natural rate of unemployment depends primarily on the level of
aggregate demand.
b.
inflation depends primarily upon the money supply growth rate.
c. there is a tradeoff between the inflation rate and the natural rate of
unemployment.
d.
2.
All of the above are correct.
If policymakers expand aggregate demand,
a. in the long run, prices will be higher and unemployment will be
lower.
b. in the long run, prices will be higher and unemployment will be
unchanged.
3.
c.
in the long run, inflation and unemployment will be unchanged.
d.
None of the above is correct.
If the short-run Phillips curve were stable, which of the following would
be unusual?
a.
an increase in inflation and an increase in output
b.
a decrease in inflation and an increase in unemployment
Chapter 35: The Short-Run Trade-off between Inflation and Unemployment
4.
5.
c.
an increase in both inflation and unemployment
d.
an increase in output and a decrease in unemployment
252
Suppose that a central bank increases the money supply. In the short-run,
according to the Phillips curve, this should make
a.
prices, output, and employment rise.
b.
prices and output rise, employment fall.
c.
prices rise and output and employment fall.
d.
prices fall, output, and employment rise.
Which of the following would we NOT expect if government policy moved
the economy up along a given short-run Phillips curve?
a. Ravi reads in the newspaper that the central bank raised the money
supply.
b.
Tony gets more job offers.
c.
Louis makes smaller increases in the prices at his health food store.
d.
All of the above are correct answers.
Chapter 35: The Short-Run Trade-off between Inflation and Unemployment
253
Figure 1
6.
7.
8.
Refer to Figure 1. If the economy starts at c and 1, then in the short run,
an increase in taxes moves the economy to
a.
b and 2.
b.
d and 3.
c.
e and 2.
d.
None of the above is correct.
In the long run, if the Fed increases the rate at which it increases the
money supply,
a.
inflation will be higher.
b.
unemployment will be lower.
c.
real GDP will be higher.
d.
All of the above are correct.
If efficiency wages became more common,
Chapter 35: The Short-Run Trade-off between Inflation and Unemployment
254
a. both the long-run Phillips curve and the long-run aggregate supply
curve would shift right.
b. both the long-run Phillips curve and the long-run aggregate supply
curve would shift left.
c. the long-run Phillips curve would shift right, and the long-run
aggregate supply curve would shift left.
d. the long-run Phillips curve would shift left, and the long-run
aggregate supply curve would shift right.
9.
The position of the long-run Phillips curve and the long-run aggregate
supply curve both depend on
a.
the natural rate of unemployment and monetary growth.
b.
the natural rate of unemployment, but not monetary growth.
c.
monetary growth, but not the natural rate of unemployment.
d.
neither monetary growth nor the natural rate of unemployment.
Figure 2
Chapter 35: The Short-Run Trade-off between Inflation and Unemployment
255
10. Refer to Figure 2. An increase in aggregate demand moves the economy
from c and 3 to
a.
a and 1 in the short run, b and 2 in the long run.
b.
b and 2 in the short run, a and 1 in the long run.
c.
d and 4 in the short run, e and 5 in the long run.
d.
None of the above is correct.
11. Refer to Figure 2. The economy would move from c and 3 to b and 2
a.
in the short run if money supply growth increased unexpectedly.
b.
in the short run if money supply growth decreased unexpectedly.
c.
in the long run if money supply growth increases.
d.
in the long run if money supply growth decreases.
12. The analysis of Friedman and Phelps can be summarized in the following
equation where a is positive number:
a. Unemployment Rate = Natural Rate of Unemployment – a(Actual
Inflation – Expected Inflation).
b. Unemployment Rate = Natural Rate of Unemployment – a(Expected
Inflation – Actual Inflation).
c. Unemployment Rate = Expected Rate of Inflation – a(Actual
Inflation – Expected Inflation).
d. Unemployment Rate = Actual Rate of Inflation – a(Actual
Unemployment – Expected Unemployment).
Chapter 35: The Short-Run Trade-off between Inflation and Unemployment
256
13. In recent years, inflation expectations have fallen. This has shifted the
short-run Phillips curve
a. left, meaning that at any given inflation rate unemployment will be
lower in the short run than before.
b. right, meaning that at any given inflation rate unemployment will be
lower in the short run than before.
c. right, meaning that at any given inflation rate unemployment will be
higher in the short run than before.
d. left, meaning that at any given inflation rate unemployment will be
higher in the short run than before.
14. Suppose a war disrupts the supply of oil to the country. We would expect
the
a. short-run aggregate supply curve, short-run Phillips curve to shift
left.
b. short-run aggregate supply curve, short-run Phillips curve to shift
right.
c. short-run aggregate supply curve to shift left, and the short-run
Phillips curve to shift right.
d. short-run aggregate supply curve to shift left, the short-run Phillips
curve to shift right
15. Suppose an economy with high inflation decides to decrease the money
supply growth rate.
a. Initially unemployment rises. Eventually the short-run Phillips curve
shifts right.
Chapter 35: The Short-Run Trade-off between Inflation and Unemployment
257
b. Initially unemployment rises. Eventually the short-run Phillips curve
shifts left.
c. Initially unemployment falls. Eventually the short-run Phillips curve
shifts right.
d. Initially unemployment falls. Eventually the short-run Phillips curve
shifts left.
16. If a central bank reduced inflation by 3 percentage points and that made
output fall by 3 percentage points each year for 3 years and the
unemployment rate rises from 3 percent to 9 percent for three years, the
sacrifice ratio is
a.
1.
b.
2.
c.
3.
d.
None of the above is correct.
ANSWER AND EXPLANATION
1. ANSWER: b. inflation depends primarily upon the money supply growth rate.
(Explain:
A is wrong. In the long-run, the natural rate of unemployment depends on real
variables (according to the classical dichotomy)
B is correct. When government prints more money, the value of each dollars
decreases => Inflation rate increases.
Chapter 35: The Short-Run Trade-off between Inflation and Unemployment
258
C is wrong. The trade-off between inflation rate and unemployment rate is only
in the short-run)
2. ANSWER: b. in the long run, prices will be higher and unemployment will be
unchanged.
(Explain:
When the Aggregate Demand curve shifts from point A to point B, unemployment rate
decreases and Inflation rate increases => Price will be higher)
3. ANSWER: c. an increase in both inflation and unemployment
(Explain: The short-run Phillips curve is downward-sloping, so if Unemployment Rate
increases, Inflation rate must decrease and vice versa
Chapter 35: The Short-Run Trade-off between Inflation and Unemployment
259
)
4. ANSWER: a. prices, output, and employment rise.
(Explain: When the central bank increases the money supply, people will demand for
more money in the money market. Thus shifting the Aggregate Demand curve to the
right, increasing the price level and Quantity of output. In the short-run Phillips curve,
employment rate will rise (Unemployment rate decreases)
)
5. ANSWER: c. Louis makes smaller increases in the prices at his health food store.
Chapter 35: The Short-Run Trade-off between Inflation and Unemployment
260
(Explain: When government policy moves the economy up along a short-run Phillips
curve, Inflation rate will rise and Unemployment rate will decrease.
A is a wrong answer. When the central bank raises money supply, Inflation rate
will increase.
B is a wrong answer. Unemployment rate decreases => People will get more job
offer.
C is a correct answer. When Inflation rate increases, so Louis has to make a
larger increase in his price.)
6. ANSWER: b. d and 3.
(Explain: The first model in the right is the model of Aggregate Demand and Supply.
When the government increases taxes, Aggregate Demand curve shift to the left => The
Chapter 35: The Short-Run Trade-off between Inflation and Unemployment
261
economy moves to point d, thus decreasing price level and inflation rate =>
Unemployment rate increases => The economy moves to point 3 in the short-run
Phillips curve)
7. ANSWER: a.
inflation will be higher.
(Explain: In the long-run, increase in money supply only affects nominal variables. So
it will not affect Unemployment rate, real GDP. However, an increase in money supply
only increases Inflation rate)
8. ANSWER: c. the long-run Phillips curve would shift right, and the long-run
aggregate supply curve would shift left.
(Explain: Efficiency wages is increasing wages above equilibrium in order to increase
the productivity of workers, creating a surplus of labor. If efficiency wages become
more common, firms will have to fire workers with less skills, thus natural rate of
unemployment will increase, long-run Phillips curve will shift right. When
unemployment increases, there will be less workers joining in production process =>
Aggregate Supply decreases => Aggregate Supply shifts left.)
9. ANSWER: b. the natural rate of unemployment, but not monetary growth.
(Explain: The determinant of the long-run Phillips curve is the natural rate of
unemployment. It is also the determinant of Aggregate Supply curve: When
Unemployment rate is low, higher quantity of products will be made, Aggregate Supply
increases at every price level and vice versa.
Chapter 35: The Short-Run Trade-off between Inflation and Unemployment
262
Monetary growth, on the other hand, is not the determinant of long-run Phillips
curve. Change in money supply will not affect Inflation Rate in the long-run.
Change in money supply leads to changes in Aggregate Demand, not Aggregate
Supply curve.)
10. ANSWER: c. d and 4 in the short run, e and 5 in the long run.
(Explain: From the equilibrium point c, a right-ward shift of Aggregate Demand will
move the equilibrium to point d in the short-run, short-run Phillips curve moves to
point 4.
In the long-run, people will adjust the expected inflation rate. This shifts the
Aggregate Supply curve to the left, moving the equilibrium from point to e. As
expected inflation rate increases, Short-run Phillips curve shifts to the right, resulting a
new equilibrium at point 5.)
11. ANSWER: b. in the short run if money supply growth decreased unexpectedly.
(Explain: In order for the equilibrium point moves from c to b, and from point 3 to 2,
Aggregate Demand curve must shift to the left. A decrease in money supply would shift
the Aggregate Demand curve to the left.)
12. ANSWER: a. Unemployment Rate = Natural Rate of Unemployment – a(Actual
Inflation – Expected Inflation).
(Explain: Theoretical Information)
Chapter 35: The Short-Run Trade-off between Inflation and Unemployment
263
13. ANSWER: a. left, meaning that at any given inflation rate unemployment will be
lower in the short run than before.
(Explain: When expected inflation drops, short-run Phillips curve will shift to the left.
Decreasing Unemployment rate at any Inflation rate given)
14. ANSWER: d. short-run aggregate supply curve to shift left, the short-run Phillips
curve to shift right
(Explain: Supply shock cause the Aggregate Supply curve to shift to the left, resulting
in lower quantity of output and higher price level =>Short-run Phillips curve shifts
right.)
15. ANSWER: b. Initially unemployment rises. Eventually the short-run Phillips curve
shifts left.
(Explain: When an economy decide to decrease money supply, Inflation rate will be
lower => Unemployment rate will be higher. In the long-run, people will adjust their
expected inflation rate. When expected inflation rate is lower, Short-run Phillips curve
shifts left)
16. ANSWER: c. 3.
(Explain: Sacrifice Ratio =
𝑇𝑜𝑡𝑎𝑙 𝑂𝑢𝑡𝑝𝑢𝑡 𝑙𝑜𝑠𝑡
𝑇𝑜𝑡𝑎𝑙 𝑑𝑒𝑐𝑟𝑒𝑎𝑠𝑒 𝑖𝑛 𝐼𝑛𝑓𝑙𝑎𝑡𝑖𝑜𝑛 𝑟𝑎𝑡𝑒
=
3×3
3
= 3)
264
Reference
Mankiw, N. G. (2015). Principles of Economics (7th ed).
Téléchargement